Ultrasound Flashcards

1
Q

Cholelithiasis

A

Cholelithiasis is the presence of a gallbladder stone or stones, without associated inflammation.

The classic clinical presentation of symptomatic cholelithiasis is colicky pain after eating a fatty meal, but it is common to see gallstones incidentally in asymptomatic patients.

Risk factors for developing gallstones include female sex, obesity, pregnancy, middle age, and diabetes.

The ultrasound diagnosis of gallstones is usually straightforward. Stones are echogenic with posterior acoustic shadowing are are often mobile. It is often helpful to reposition the patient (typically in the left lateral decubitus position) while scanning to assess whether the stones layer dependently to differentiate stones from polyps or other masses.

A gallbladder completely filled with stones can be more challenging to identify. The wall-echo-shadow (WES) signn describes the appearance of a gallbladder full of multiple stones (or one giant stone).

Two parallel echogenic arcs represent the gallbladder wall and leading edge of the stone, with an intervening thin layer of hypoechoic bile. The gallstone typically casts a prominent shadow.

The differential diagnosis of echogenic material within the gallbladder includes:

Gallstones (mobile shadowing)

Gallbladder sludge (mobile, non-shadowing)

Gallbladder polyp (non-mobile, non-shadowing, often attached to the gallbladder wall via a stalk, may be vascular).

Hyperplastic cholecytoses (non-mobile, multiple polyps).

How well did you know this?
1
Not at all
2
3
4
5
Perfectly
2
Q

Acute calculous cholecystitis

A

Acute cholecystitis is inflammation of the gallbladder, usually due to a gallstone impacting the cystic duct. Ultrasound is the first-line evaluation of suspected acute cholecystitis.

Acute cholecystitsi clinicaly presents with right upper quadrant (RUQ) pain and fever.

There is no 100% specific ultrasound finding for acute cholecystitis. However, gallstones are seen >90% of the time and a postivie sonographic Murphy’s sign (RUQ pain with pressure from the transducer) also has a high positive predictive value. Other findings include: Gallbladder wall thickening >3 mm. Distended gallbladder >4 cm in diameter. Pericholecystic fluid. Color Doppler showing hyperemic gallbladder wall. Hyperechoic fat in the gallbladder fossa (ultrasound correlate to CT finding of fat stranding).

Complications of acute cholecystitis are rare but serious. Emphysematous cholecystitis is gas in the gallbladder wall and has a high risk of gallbladder perforation. Gangrenous cholecystitis is necrosis of the gallbladder wall. Sonographic findings include layering echogenic material in the gallbladder lumen representing hemorrhage and sloughed membranes. Gallbladder perforation appears as focal discontinuity of the gallbladder wall. Perihepatic ascites containing dirty echoes is often present.

Surgical treatment of uncomplicated acute calculous cholecystitis is cholecystectomy. In patients who are not good surgical candidates, a temporizing percutaneous cholecystostomy tube can be placed prior to definitive surgical cholecystectomy.

How well did you know this?
1
Not at all
2
3
4
5
Perfectly
3
Q

Acalculous cholecystitis

A

Acalculous cholecystitis is cholecystitis without gallstones, typically seen in very sick patients. Risk factors include sepsis, prolonged total parenteral nutrtion, and trauma.

The ultrasound appearance is similar to acute cholecystitis but without stones. Since many patients are ventilated or obtunded, it’s often not possible to evaluate for sonographic Murphy’s sign.

Treatment of acalculous cholecystitis is typically interventional radiology percutaneous cholecystostomy. Unlike the treatment of calculous cholecystitis, cholecystostomy is often the definitive therapy.

How well did you know this?
1
Not at all
2
3
4
5
Perfectly
4
Q

Emphysematous cholecystitis

A

Emphysematous cholecystitis is a rapidly progressive form of acute cholecystitis characterized by gas in the gallbladder wall. Emphysematous cholecystitis is associated with gallbladder ischemia causing bacterial translocation. Treatment is urgent surgery.

On ultrasound, gas is usually present in both the gallbladder lumen and wall, which appears as echogenic lines and foci with posterior dirty shadowing.

How well did you know this?
1
Not at all
2
3
4
5
Perfectly
5
Q

Porcelain gallbladder

A

A porcelain gallbladder is a calcified gallbladder wall due to either chronic irritation from supersaturated bile or repeated bouts of gallbladder obstruction.

Porcelain gallbladder is associated with an increased risk of gallbladder cancer, but the incidence is controversial. In general, prophylactic choleycystectomy is the standard of care.

On ultrasound, the wall of the gallbladder is echogenic, and there are almost always associated gallstones.

The differential diagnosis of an echogenic gallbladder wall includes a porcelain gallbladder, a gallbladder packed full of stones (which will feature the wall-echo-shadow sign), or emphysematous choleycystitis (intramural gas will have dirty shadowing).

How well did you know this?
1
Not at all
2
3
4
5
Perfectly
6
Q

Courvoisier gallbladder

A

The Courvoisier gallbladder refers to a markedly dilated gallbladder (originally described as being so large as to be directly palpable) from malignant obstruction of the common bile duct.

A markedly distended gallbladder implies chronic obstruction of either the cystic duct (when seen in isolation) or the common bile duct (when seen in combination with dilation of the common bile duct and intrahepatic biliary dilation).

How well did you know this?
1
Not at all
2
3
4
5
Perfectly
7
Q

Overview of hyperplastic cholecytoses

A

The hyperplastic cholecytoses are a spectrum of non-neoplastic proliferative disorders caused by deposition of cholesterol-laden macrophages within the wall of the gallbladder. The cholecystoses range from abnormalities of the gallbladdr wall (adenomyomatosis and strawberry gallbladder) to gallbladder polyps extending into the lumen.

How well did you know this?
1
Not at all
2
3
4
5
Perfectly
8
Q

Adenomyomatosis

A

Adenomyomatosis is cholesterol deposition in mural Rokitansky-Aschoff sinuses. It is important not to confuse with adenomyosis of the uterus: It may be helpful to remember that there are three L’s in gallbladder, and adenomyomatosis is a longer word than adenomyosis.

The ultrasound hallmark of adenomyomatosis is the comet-tail artifact due to reflections off of tiny crystals seen in a focally thickened and echogenic gallbladder wall.

How well did you know this?
1
Not at all
2
3
4
5
Perfectly
9
Q

Strawberry gallbladder (cholesterolosis of the gallbladder)

A

Strawberry gallbladder is a pathologic diagnosis that is not apparent by imaging. It is characterized by tiny mural cholesterol depsosits likened to strawberry seeds.

How well did you know this?
1
Not at all
2
3
4
5
Perfectly
10
Q

Gallbladder polyps

A

Most gallbladder polyps are benign cholesterol polyps that are part of the hyperplastic cholecytosis spectrum. Rarely (<5%), polyps may be premalignant adenomas.

Clinically, gallbladder polyps may cause right upper quadrant pain or even cholecystitis if the cystic duct is obstructed.

The following characteristics, known as the six S’s, increase the risk for a polyp being malignant: Size >10 mm or rapid growth. As a caveat, ultrasound has limited sensitivity and specificity in detecting small polyps (<10 mm), especially in the presence of gallstones. Single: A solitary polyp is more suspicious for malignancy. In contrast, benign cholesterol polyps tend to be multiple. Sessile (broad-based): Sessile morphology is suspicious. A polyp is more likely benign if pedunculated. Stones: The presence of stones may induce chronic inflammation, which can predispose towards malignancy. Primary Sclerosing cholangitis increases risk of malignancy. Sixty (age) or greater.

In patients with several of these high-risk features, cholecystectomy should be considered in the presence of a polyp greater than 6 mm in size.

The typical ultrasound appearance of a polyp is a non-mobile, non-shadowing polypoid lesion extending from the wall into the lumen of the gallbladder. There may be vascular flow in the stalk.

The main differential consideration is adherent sludge, which will not have any vascular flow.

How well did you know this?
1
Not at all
2
3
4
5
Perfectly
11
Q

Primary gallbladder carcinoma

A

Gallbladder cancer is a rare malignancy with a poor prognosis. A typical clinical presentation may include right upper quadrant pain, weight loss, and jaundice.

Risk factors for development of gallbladder cancer include: Gallstones and chronic cholecystitis. Porcelain gallbladder (somewhat controversial). Primary sclerosing cholangitis. Inflammatory bowel disease (ulcerative colitis more frequently than Crohn disease). Adenomatous polyp >10 mm or >6 mm with multiple risk factors, as described above.

Ultrasound shows a polypoid mass with increased vascularity in the gallbladder. There is often direct invasion into the liver. Regional adenopathy occurs early. Bile duct obstruction may be present.

How well did you know this?
1
Not at all
2
3
4
5
Perfectly
12
Q

Gallbladder metastases

A

Metastases to the gallbladder are uncommon.

Hepatocellular carcinoma can spread directly to the gallbladder through the bile ducts.

Melanoma can spread hematogenously to the gallbladder mucosa.

How well did you know this?
1
Not at all
2
3
4
5
Perfectly
13
Q

Diffuse gallbladder wall thickening >3 mm (most common causes)

A

Fluid-overload/edematous states: Cirrhosis - Hypoalbuminemia leads to diffuse gallbladder wall thickening. Congestive heart failure. Protein-wasting nephropathy.

Inflammatory/infectious: Cholecystitis, usually with associated cholelithiasis. Hepatitis. Pancreatitis. Diverticultitis.

Infiltrative neoplastic disease: Gallbladder carcinoma. Metastases to gallbladder (rare).

Post-prandial state.

How well did you know this?
1
Not at all
2
3
4
5
Perfectly
14
Q

Focal Gallbladder wall thickening (common causes)

A

Hyperplastic cholecystoses: Adenomyomatosis and cholesterol polyp.

Vascular: Varices.

Neoplastic disease: Adenomatous polyp. Gallbladder carcinoma. Adjacent hepatic tumor.

How well did you know this?
1
Not at all
2
3
4
5
Perfectly
15
Q

Non-shadowing “mass” in the gallbladder lumen

A

Tumefactive sludge (mobile).

Blood/pus (mobile).

Gallbladder polyp (immobile).

Gallbladder carcinoma (immobile).

How well did you know this?
1
Not at all
2
3
4
5
Perfectly
16
Q

Echogenic gallbladder wall

A

Porcelain gallbladder

Gallbladder full of stones (signified by the wall-echo-shadow sign).

Emphysematous cholecystitis.

How well did you know this?
1
Not at all
2
3
4
5
Perfectly
17
Q

Choledocholithiasis

A

Choledocholithiasis is a stone in the common bile duct, generally treated with ERCP.

How well did you know this?
1
Not at all
2
3
4
5
Perfectly
18
Q

Mirizzi syndrome

A

Mirizzi syndrome is seen when a stone in the cystic duct causes inflammation and external compression of the adjacent common hepatic duct (CHD).

Essential for the surgeon to know about preoperatively because the CHD may be mistakenly ligated instead of the cystic duct. Additionally, inflammation can cause the gallstone to erode into the CHD and cause a cysto-choledochal fistula and biliary obstruction.

On ultrasound, a stone is typically impacted in the distal cystic duct, and the CHD is dilated. The cystic duct tends to run in parallel with the CHD.

How well did you know this?
1
Not at all
2
3
4
5
Perfectly
19
Q

Pneumobilia

A

Pneumobilia is air in the biliary tree. It is commonly seen after biliary interventions, but may be due to cholecystoenteric fistula or rarely emphysematous cholecystitis.

On ultrasound, small echogenic gas bubbles are seen centrally in the liver with posterior dirty shadowing.

In contrast to pneumobilia, portal venous gas (which implies bowel ischemia until proven otherwise) is peripheral and causes a spiky appearance of the portal vein spectral Doppler waveform.

How well did you know this?
1
Not at all
2
3
4
5
Perfectly
20
Q

Cholangiocarcinoma

A

Cholangiocarcinom is cancer of the bile ducts. It classically presents with painless jaundice. Most cases of cholangiocarcinoma are sporadic, although key risk factors include chronic biliary disease (in the US) and liver fluke infection (in the Far East).

The hilum is the most common location of cholangiocarcinoma. A hilar cholangiocarcinoma is known as a Klatskin tumor. Intrahepatic cholangiocarcinoma occurs uncommonly (10%).

Ultrasound plays a role in the initial evaluation of adjacent adenopathy and vascular structures. Local nodes include porta hepatis and hepatoduodenal ligament nodes. If more distal nodal disease is present, then the tumor is generally considered unresectable.

How well did you know this?
1
Not at all
2
3
4
5
Perfectly
21
Q

Hepatic Steatosis

A

Hepatic steatosis is the accumulation of excess fat within hepatocytes due to a metabolic derangement (obesity or diabetes), hepatotoxins (EtOH), or prolonged fasting.

Ultrasound shows a diffuse increase in hepatic echogenicity. Normally, the liver and kidney should have the same echogenicity. With fatty infiltration, the liver appears more echogenic than the kidney. Hepatic steatosis also causes increased sound attenuation, leading to poor visualization of deeper structures.

Focal fat sparing is a geographic area of hypoechogenicity in an otherwise fatty liver. A characteristic location of focal fat sparring is the gallbladder fossa.

How well did you know this?
1
Not at all
2
3
4
5
Perfectly
22
Q

Cirrhosis

A

Cirrhosis is the replacement of functioning hepatocytes with dysfunctional fibrotic tissue, due to long-standing repeated cycles of hepatocyte injury and repair.

Micronodular cirrhosis causes cirrhotic nodules less than 3 mm in size and is most commonly associated with alcoholism.

Macronodular cirrhosis features larger nodules (>3 mm) separated by wide scars and fibrous septae. Macronodular cirrhosis is caused by fulminant viral hepatitis which does not uniformly affect the liver.

The typical utrasound appearance of cirrhosis is a coarse, heterogeneously increased liver echotexture with a nodular external contour. In early cirrhosis, the superficial nodularity is best appreciated with a high frequency linear probe. The caudate lobe is often spared and hypertrophies in response to increased demand (the caudate has direct venous drainage into the IVC and therefore can bypass the hypertensive portal system). End-stage cirrhosis is characterized by a shrunken, nodular liver.

Signs of portal hypertension are often present, including an enlarged portal veins, splenomegaly, varices, portosystemic shunts, and a patent umbilical vein. Imaging of portal hypertension is discussed in detail in the liver Doppler section.

How well did you know this?
1
Not at all
2
3
4
5
Perfectly
23
Q

Viral hepatitis

A

Viral hepatitis is infection of the liver by a hepatotropic virus. Hepatitis B and C cause chronic disease.

The most common ultrasound finding is a normal liver. Occasionally periportal edema produces the characteristic starry sky pattern of increased portal triad echogenicity.

Acute hepatitis is often associated with diffuse severe gallbladder wall thickening.

How well did you know this?
1
Not at all
2
3
4
5
Perfectly
24
Q

Pyogenic abscess

A

Pyogenic abscess is caused by pus-forming organisms and is usually due to spread from intestinal or biliary infection (most commonly E. coli)

Infection starts as an ill-defined area of altered echogenicity (phlegmon stage) that evolves into a well-defined hypoechoic structure with internal echoes (mature abscess).

How well did you know this?
1
Not at all
2
3
4
5
Perfectly
25
Q
A
How well did you know this?
1
Not at all
2
3
4
5
Perfectly
26
Q

Amebic abscess

A

Amebic abscess is caused by Entamoeba histolytica. A near-universal presenting symptom is pain, seen in 99% of patients. The most common location is near the dome of the right lobe.

On ultrasound, an amebic abscess is indistinguishable from a pyogenic abscess and appears as a hypoechoic structure with low-level internal echoes.

Antimicrobial therapy is usually sufficient treatment, and drainage is rarely necessary.

How well did you know this?
1
Not at all
2
3
4
5
Perfectly
27
Q

Echinococcal cyst (hydatid disease)

A

Echinococcal cyst is caused by larvae of Echinococcus granulosus, most commonly found in endemic areas in the middle east, Mediterranean, and South America.

There is risk of anaphylaxis with peritoneal spillage of cyst fluid, although these are often biopsied and drained uneventfully. Medical treatment is albendazole or mebendazole.

Classic ultrasound appearance is a large liver cyst with numerous peripheral daughter cysts. A highly suggestive finding is the change in position of daughter cysts as the patient is repositioned.

The water-lily sign is an undulating membrane within the hydatid cyst.

Hydatid sand is a fine sediment caused by seperation of the membranes from the endocyst.

How well did you know this?
1
Not at all
2
3
4
5
Perfectly
28
Q

Candidiasis

A

Hepatic candidiasis is a rare infection in the immunocompromised due to Candida albicans or Candida glabrata.

On imaging, there are multiple tiny targetoid lesions. The presence of concurrent similar-appearing lesions in the spleen is highly suggestive of hepatosplenic candidiasis.

How well did you know this?
1
Not at all
2
3
4
5
Perfectly
29
Q

Hepatic Pneumocystis jiroveci

A

Hepatic Pneumocystis jiroveci is seen in disseminated disease in the severly immunocompromised. Hepatic infection is classically secondary to the use of inhaled pentamidine to treat Pneumocystis pneumonia, as pentamidine is not absorbed systemically and thus would not prevent hepatic infection.

Ultrasound shows multiple punctate echogenic calcifications in the liver and often spleen.

How well did you know this?
1
Not at all
2
3
4
5
Perfectly
30
Q

Cavernous hemangioma

A

Hepatic cavernous hemangioma is the most common benign hepatic neoplasm.

The classic ultrasound appearance of hemangioma is a solitary, circumscribed, homogenously echogenic mass with no flow on color Doppler. Posterior acoustic enhancement is nonspecific but may be present. When see, posterior acoustic enhancement is thought to correlate with hypervascularity. A hypoechoic halo should never be seen - this finding suggests malignancy.

Hepatic hemangioma can rarely have an atypica hypoechoic appearance when seen in a fatty liver.

If a solitary classic-appearing hemangioma is seen and the patient hasj an otherwise normal-appearing liver, normal LFTs, no known malignancy, and is asymptomatic, then no further workup is required.

Any hererogeneity or atypical ultrasound findings should prompt consideration of an alternative diagnosis. The differential of a hyperechoic hepatic mass includes hyperechoic hepatocellular carcinoma or metastatic disease (even in the absence of a halo). In a patient with cirrhosis or any known primary malignancy, further workup (MRI or CT) is usually warranted if the mass is new, even if classic appearing.

How well did you know this?
1
Not at all
2
3
4
5
Perfectly
31
Q

Focal Nodular Hyperplasia (FNH)

A

Focal nodular hyperplasia (FNH) is a benign hyperplastic hepatic mass with a central non-fibrotic stellate scar consisting of biliary ductules and venules.

Ultrasound findings are nonspecific. The central scar is rarely seen on ultrasound, and even when it is, this finding can be seen in other lesions, including hepatocellular carcinoma, giant hemangioma, or adenoma.

FNH is often difficult ot detect on sonography. It may be nearly isoechoic to normal liver and manifest on imaging as a subtle displacement of the hepatic contour.

Doppler findings of FHN include a spoke-wheel configuration of arterial vessels.

MRI or Tc-99m sulfur colloid scintigraphy can confirm (classically, FNH has increased uptake of sulfur colloid). MRI is by far the more useful test.

How well did you know this?
1
Not at all
2
3
4
5
Perfectly
32
Q

Hepatic Adenoma

A

Hepatic adenoma is a benign liver tumor associated with oral contraceptives, anabolic steroids, and type I glycogen storage disease (von Gierke’s disease - in which case adenomas will be multiple).

Due to high incidence of hemorrhage, adenomas are usually resected.

There are no specific ultrasound features to distinguish an adenoma from other hepatic masses. An adenoma may be hyperechoic, isoechoic, or hypoechoic relative to normal liver.

Adenoma is usually photopenic on Tc-99m sulfur colloid scintigraphy (in contrast to FNH).

How well did you know this?
1
Not at all
2
3
4
5
Perfectly
33
Q

Hepatic lipoma

A

Hepatic lipoma is a benign neoplasm composed of fat that appears as a well-defined hyperechoic mass. It may appear identical to hemagioma or hyperechoic hepatocellular carcinoma.

When multiple, may be associated with tuberous sclerosis and renal angiomyolipomas.

How well did you know this?
1
Not at all
2
3
4
5
Perfectly
34
Q

Biliary cystadenoma

A

Biliary cystadenoma is a benign cystic mass lined with biliary-type epithelium.

Although benign, most are surgically resected since malignant transformation may occur.

Biliary cystadenoma appears as a multiseptated cystic mass on all imaging modalities. Mural nodules should be regarded with suspicion. The presence of mural nodularity suggests malignant transformation to cystadenocarcinoma.

How well did you know this?
1
Not at all
2
3
4
5
Perfectly
35
Q

Hepatic metastases

A

Metastatic disease to the liver is far more common than primary hepatocellular carcinoma.

Metastases can have a variable ultrasound appearacne, although the classic finding is a hypoechoic rim producing a target sign.

Hypoechoic hepatic metastases include: Breast (can be either hypoechoic or hyperechoic). Pancreas. Lung. Lymphoma.

Hyperechoic hepatic metastases include: Colon cancer is hyperechoic in greater than 50% of cases. A hyperechoic appearance may suggest a better prognosis. Renal cell carcinoma. Breast (can be either hyperechoic or hypoechoic). Carcinoid. Choriocarcinoma.

Calcified hepatic metastases (hyperechoic with acoustic shadowing) include: Colon cancer (especially mucinous type). Gastric adenocarcinoma. Osteosarcoma (very rare).

Cystic hepatic metasteses include: Ovarian cystadenocarcinoma. Gastrointestinal sarcoma.

Infiltrative metastases include: Lung. Breast. In particular, treated breast cancer may cause a pseudo-cirrhosis appearance. Prostate.

How well did you know this?
1
Not at all
2
3
4
5
Perfectly
36
Q

Hepatocellular carcinoma (HCC)

A

Hepatocellular carcinoma (HCC) is a hepatic malignancy arising in teh setting of chronic inflammation.

Patients with cirrhosis or chronic viral hepatitis are regularly screened for HCC with serum alpha-fetoprotein levels and ultrasound. Ultrasound is not very sensitive to detect small HCC in end-stage cirrhotic livers.

HCC has a variety of ultrasound appearances - therefore, a mass in a cirrhotic liver is considered HCC until proven otherwise. High Doppler flow may be present, especially at the periphery of the mass, due to arteriovenous shunting.

HCC has a propensity for venous invasion. The portal veins should always be carefully evaluated in the presence of a hepatic mass. Internal Doppler flow within a venous clot suggests a tumor thrombus.

How well did you know this?
1
Not at all
2
3
4
5
Perfectly
37
Q

Fibrolamellar carcinoma

A

Fibrolamellar carcinoma is a variant of HCC seen in young adults without cirrhosis and is not associated with elevated alpha-fetoprotein.

Fibrolamellar carcinoma has a much better prognosis compared to typical HCC.

How well did you know this?
1
Not at all
2
3
4
5
Perfectly
38
Q

Hepatic lymphoma

A

Primary hepatic lymphoma may present as a single mass or multiple masses.

Lymphoma tends to be hypoechoic and may demonstrate the target sign typical of metastases.

How well did you know this?
1
Not at all
2
3
4
5
Perfectly
39
Q

Post-transplant lymphoproliferative disorder (PTLD)

A

Post-transplant lymphoproliferative disorder (PTLD) is a type of lymphoma caused by Epstein-Barr virus that arises after solid organ or bone marrow transplant. Patients with renal transplants are at particular risk for development of PTLD. PTLD may occur anywhere, regardless of which organ was transplanted.

Treatment is reduction/withdrawal of immunosuppression.

PTLD appears as a mass with a variable and nonspecific ultrasound appearance. Therefore, it is important to mention PTLD if a liver mass is seen in a transplant patient.

How well did you know this?
1
Not at all
2
3
4
5
Perfectly
40
Q

Multicystic liver

A

Multiple simple cysts.

Caroli disease (saccular dilation of the intrahepatic bile ducts).

Autosomal dominant polycystic kidney disease (ADPKD): Liver cysts seen in >50% of patients.

How well did you know this?
1
Not at all
2
3
4
5
Perfectly
41
Q

Liver cyst with internal echoes

A

Simple cyst with internal hemorrhage.

Liver abscess

Hematoma.

Necrotic or cystic metastasis (ovarian cystadenocarcinoma or gastrointestinal sarcoma).

How well did you know this?
1
Not at all
2
3
4
5
Perfectly
42
Q

Multiple echogenic liver lesions

A

Prior granulomatous exposure.

Disseminated pneumocystis in AIDS. Classic history is treatment with inhaled pentamidine, which does not have systemic absorption.

How well did you know this?
1
Not at all
2
3
4
5
Perfectly
43
Q

Portal hypertension

A

Portal hypertension is increased pressure of the portal venous system. It can be classified in relation to the hepatic capillary bed as pre-sinusoidal, sinusoidal, or post-sinusoidal: Pre-sinusoidal: Insult is proximal to the hepatic parenchyma, such as portal vein thrombosis. Sinsusoidal: Insult is hepatic in origin, such as cirrhosis. Post-sinusoidal: Insult is beyond the liver, such as Budd-Chiari (heptic vein thrombosis) or IVC thrombosis.

Normally, the portal veins and hepatic arteries flow in the same direction, toward the liver. This direction is called hepatopedal flow (-petal =toward). The normal portal venous waveform is above the baseline (hepatopetal) and gently undulating.

A pulsatile portal venous waveform is abnormal. The differential diagnosis for a pulsatile portal venous waveform includes tricuspid regurgitation and right-sided CHF. The differential diagnosis for hepatic vein pulsatility is similar, and is discussed in the following section.

Portal pressure is defined as a direct portal venous pressure of >5 mm Hg, although the portal venous pressure is not measured directly.

Ultimately, when portal venous pressure is higher than forward pressure, the portal venous flow will reverse, which is diagnostic for portal hypertension. Reversal of portal venous flow is called hepatofugal flow (-fugal = away, same Latin root as fugitive).

In addition to flow reversal, there are several secondary findings of portal hypertension: Low portal venous velocity (<16 cm/sec) Dilated portal vein (13 mm is the maximal normal diameter in quiet respiration). Splenomegaly. Varices. Portosystemic shunts are often present, most commonly gastro-esophageal, paraumbilical, or splenorenal. Note that an isolated portosystemic shunt may not be caused by portal hypertension. For instance, isolated obstruction of the splenic vein from pancreatitis or neoplasm may lead to a shunt. A recanalized umbilical vein is a portosystemic shunt that is diagnostic of portal hypertension.

How well did you know this?
1
Not at all
2
3
4
5
Perfectly
44
Q

Transjugular intrahepatic portosystemic shunt (TIPS)

A

Portal hypertension (and reversal of portal flow) can be treated with a transjugular intrahepatic portosystemic shunt (TIPS), which connects a branch of the portal vein to a systemic hepatic vein.

Ultrasound is used for surveillance of TIPS patency, starting with a post-procedure baseline. Routine follow-up is performed according to the following schedule: In 1 month, every 3 months for the first year, and then every 6-12 months.

Flow in a patent TIPS will be towards the hepatic veins, and flow in the portal veins will be towards the TIPS. Therefore, flow in the main portal vein will be hepatopetal and flow in the right and left portal vein will be hepatofugal (highlighted below with yellow circles).

If the TIPS becomes occluded, the right and left portal veins will “re-reverse” and become hepatopetal.

How well did you know this?
1
Not at all
2
3
4
5
Perfectly
45
Q

Portal vein thrombosis

A

Thrombosis of the portal vein can be bland (simple thrombosis) or may be due to tumor invasion.

Bland portal vein thrombosis can be caused by general hypercoagulable state or may be due to local inflammation from pancreatitis or hepatitis. In infants, omphalitis may also lead to portal vein thrombosis.

Tumor thrombus is most commonly caused by hepatocellular carcinoma.

Ultrasound of portal vein thrombosis shows lack of portal venous flow, often with echogenic thrombus within the portal vein. Expansion of the portal vein can be seen with either bland or tumor thrombus. On color Doppler, flow within the thrombus suggests tumor thrombus.

One potential pitfall to be aware of is slow (<16 cm/sec) or stagnant portal venous flow in the presence of portal hypertension which may mimic portal vein thrombosis.

Long-standing portal vein thrombosis leads to cavernous transformation of the portal vein, characterized by formation of multiple small periportal collaterals.

How well did you know this?
1
Not at all
2
3
4
5
Perfectly
46
Q

Portal venous gas

A

Portal venous gas is due to abdominal catastrophe (ischemia and infarction) until proven otherwise. If the cause of the portal venous gas is unkown, CT should be performed emergently.

Grayscale ultrasound shows peripheral patchy branching foci of hyperechogenicity that are often transient. Spectral Doppler of the portal vein features numerous characteristic spikes.

In contrast to protal venous gas, pneumobilia tends to be more central.

How well did you know this?
1
Not at all
2
3
4
5
Perfectly
47
Q

Normal hepatic vein waveform

A

The hepatic veins feed into the IVC and the right side of the heart. The spectral Doppler waveform of the hepatic veins is therefore affected by the cardiac cycle.

The normal hepatic venous waveform has three distinct components: The A, S, and D waves. Note that antegrade flow is defined as forward flow in the normal expected direction.

How well did you know this?
1
Not at all
2
3
4
5
Perfectly
48
Q

Increased hepatic vein pulsatility: Accentuated A-wave

A

Increased hepatic vein pulsatility is caused by a right-sided cardiac abnormality, either right-sided heart failure or tricuspid regurgitation. Both conditions are characterized by accentuation of the A-wave due to increased retrograde flow during atrial systole.

Normally, the tricuspid valve closes at the beginning of ventricular systole (the beginning of the S-wave).

In tricuspid regurgitation, there is some degree of blood flow from the right ventricle into the right atrium during ventricular systole, allowing less blood to return to the right atrium from the hepatic veins and IVC during ventricular systole. This results in a decreased or even retrograde S-wave.

In right-sided heart failure, the tall A-wave is due to increased right atrial pressure; however, in contrast to tricuspid regurgitation, the S-wave is normal since the tricuspid valve remains competent.

How well did you know this?
1
Not at all
2
3
4
5
Perfectly
49
Q

Decreased hepatic vein pulsatility

A

Decreased hepatic vein pulsatility is seen in cirrhosis, Budd-Chiari (hepatic vein thrombosis), and hepatic veno-occlusive disease.

How well did you know this?
1
Not at all
2
3
4
5
Perfectly
50
Q

Pancreatitis

A

Pancreatitis is inflammation of the pancreatic parenchyma, most often caused by alcohol or gallstones.

Ultrasound is useful in the initial evaluation of pancreatitis to evaluate for gallstones or biliary obstruction.

Usually, the pancreas appears normal in acute pancreatitis. The pancreas may be diffusely enlarged and relatively hypoechoic due to edema. More severe inflammation may cause the normally hypoechoic pancreas to become isoechoic to liver.

Ultrasound has limited utility in evaluating complications of pancreatitis such as pancreatic necrosis or peripancreatic fluid collections.

Complications of acute pancreatitis may be inflammatory, infectious, or vascular: A pancreatic pseudocyst is usually detectable by ultrasound, although the full extent of large pseudocysts can be difficult ot determine by ultrasound alone.

Infectious compications of acute pancreatitis include peripancreatic abscess, infected pseudocyst, and infected pancreatic necrosis.

The two most important vascular complications of pancreatitis are splenic vein thrombosis and splenic artery pseudoaneurysm, both of which can be characterized by Doppler ultrasound.

How well did you know this?
1
Not at all
2
3
4
5
Perfectly
51
Q

Chronic pancreatitis

A

Chronic pancreatitis is caused by repeated bouts of acute pancreatitis (most commonly alcoholic).

The classic ultrasound appearance of chronic pancreatitis is an atrophied gland, with diffuse calcifications and dilated and beaded distal pancreatic duct.

Calculi within the pancreatic duct may also be seen.

How well did you know this?
1
Not at all
2
3
4
5
Perfectly
52
Q

Pancreatic adenocarcinoma

A

Pancreatic adenocarcinoma is the most common pancreatic tumor, and is typically seen in older males.

Small tumors are hypoechoic, while larger masses may be more heterogenous. It can be difficult to identify the tumor extent on ultrasound because of infiltrative margins and invasion of the tumor into pancreatic parenchyma and adjacent structures.

The most common locationfor a tumor to arise is the pancreatic head, where the mass often presents with ductal obstruction. The double duct sign represents dilation of both the pancreatic and common bile ducts caused by malignant obstruction.

How well did you know this?
1
Not at all
2
3
4
5
Perfectly
53
Q

Cystic pancreatic neoplasms

A

Cystic pancreatic neoplasms are a diverse group of unrelated pancreatic tumors that may appear similar by ultrasound.

Serous cystadenoma is a benign tumor seen in older females, consisting of multipe tiny cysts. A characteristic calcified scar is not often seen, but is very specific when present.

Mucinous cystic neoplasm has malignant potential, and is usually seen in middle-aged females. Compared to serous cystadenoma, the cysts are larger in size. The mucin can generate numerous fine echoes.

Intraductal papillary mucinous neoplasm (IPMN) is a neoplasm of variable and controversial natural history that communicates with either the main pancreatic duct or branch ducts. Demonstration of the ductal communication can be difficult by ultrasound.

How well did you know this?
1
Not at all
2
3
4
5
Perfectly
54
Q

Pancreatic endocrine tumors

A

Tumors arising from the neuroendocrine cells of the pancreas may be either functioning or nonfunctioning.

Functioning tumors are usually symptomatic, small at diagnosis, and identified through biochemical testing. In contrast, nonfunctioning tumors may be asymptomatic, and hence, large at the time of diagnosis.

Intraoperative ultrasound continues to gain ground and is helpful in identifying small tumors at surgery.

Insulinomas are the most common pancreatic tumors. Preoperative ultrasound detection is difficult and is successful less than 60% of the time. When seen, insulinomas are hypoechoic, encapsulated pancreatic nodules.

Gastrinomas are the second most common pancreatic endocrine tumors. Liver metastases are present at the time of diagnosis in 60% of patients.

How well did you know this?
1
Not at all
2
3
4
5
Perfectly
55
Q

Pancreatic lymphoma

A

B-cell lymphoma is the most common subtype of lymphoma to affect the pancreas, and is almost always associated with adenopathy, and multi-organ involvement by the time the pancreas is involved.

The typical ultrasound appearance of pancreatic lymphoma is diffusely enlarged, hypoechoic gland.

How well did you know this?
1
Not at all
2
3
4
5
Perfectly
56
Q

Splenic calcifications

A

Granulomatous disease: Calcifications may be scattered or diffuse.

Splenic infarct.

Hematoma.

Calcified splenic artery aneurysm.

How well did you know this?
1
Not at all
2
3
4
5
Perfectly
57
Q

Cystic splenic lesion: Color Doppler should always be used to exclude a vascular etiology

A

Splenic artery aneurysm or pseudoaneurysm.

Hematoma.

Abscess.

Pancreatic pseudocyst.

How well did you know this?
1
Not at all
2
3
4
5
Perfectly
58
Q

Echogenic splenic lesion

A

Hemangioma (can also be hypoechoic).

Hamartoma.

Lymphangioma.

How well did you know this?
1
Not at all
2
3
4
5
Perfectly
59
Q

Hypoechoic splenic lesion

A

Laceration (in the setting of trauma)

Abscess.

Lymphoma.

Sarcoidosis.

Metastasis.

Infarct (tends to be peripheral).

Extramedullary hematopoiesis.

How well did you know this?
1
Not at all
2
3
4
5
Perfectly
60
Q

Splenomegaly (defined as >14 cm in sagittal plane)

A

Mild to moderate splenomegaly: Portal hypertension (most common). Infection. AIDS.

Moderate to marked splenomegaly: Leukemia/lymphoma. Infectious mononucleosis.

Massive splenomegaly: Myelofibrosis.

How well did you know this?
1
Not at all
2
3
4
5
Perfectly
61
Q

Evaluation of kidney stones

A

Ultrasound is an excellent modality for evaluation of nephrolithiasis, which may cause renal obstruction and resultant hydronephrosis.

An echogenic shadowing focus in the kidney, ureter, or bladder, is suspicious for a stone.

After diagnosing a renal or ureteral calculus, one should always evaluate for the presence of hydronephosis and perinephric fluid.

How well did you know this?
1
Not at all
2
3
4
5
Perfectly
62
Q

Approach to hydronephrosis

A

The most common cause of hydronephrosis is an obstructing calculus.

Although hydronephrosis is usually due to ureteral obstruction, it is possible tohave hydronephrosis without obstruction. For instance, vesicoureteral reflux or pregnancy may cause a dilated ureter without obstruction. Pregnancy preferentially affects the right side.

Likewise, obstruction without hydronephrosis may also be seen in: Very acute obstruction. Obstruction with dehydration, where there is insufficient urine production to create a pressure backup. Obstruction with ruptured fornix. Increased pressure from obstruction may cause a fornix to rupture, which could decompress the renal pelvis and spill fluid into the perinephric space.

How well did you know this?
1
Not at all
2
3
4
5
Perfectly
63
Q

Pitfalls in diagnosing hydronephrosis

A

It can sometimes be difficult to distinguish between hydronephrosis and multiple renal sinus cysts. Renal sinus cysts are subsequently discussedand include peripelvic and parapelvic cysts. On imaging, renal sinus cysts will show a single or mulitple discrete cystic lesions that do not communicate with each other.

In true hydronephrosis, all the dilated fluid-filled spaces are contiguous.

How well did you know this?
1
Not at all
2
3
4
5
Perfectly
64
Q

Resistive index (RI) may be helpful in diagnosing obstruction

A

The renal resistive index (RI) may be elevated in acute obstruction, thought to be due to cytokine-mediated renal artery vasoconstriction.

The resitive index is calculated with pulse-width Dopper of the renal segmental or arcuate arteries. RI = (PSV - EDV)/PSV. PSV = peak systolic velocity. EDV = end diastolic velocity. Higher resistive indices. With no diastolic flow PSV/PSV = 1. Reversal of diastolic flow technically causes RI > 1, although in such cases RI is not measured.

A RI of >0.7 on the affected side, or a difference of >0.1 between kidneys suggests acute obstruction. Bilateral elevated RIs (>0.7) are nonspecific and can be due to any number of medical renal processes.

The resistive index is not used to diagnose chronic obstruction.

How well did you know this?
1
Not at all
2
3
4
5
Perfectly
65
Q

Ureteral jets may be helpful but are controversial

A

A ureteral jet is flow of urine into the bladder as seen by color Doppler.

Flow from the kidney to the bladder would be completely eliminated in complete obstruction, so theoretically the presence of a utereral jet rules out a complete obstruction. However, ureteral jets are very commonly seen even with stones, and jets are often absent in normal patients.

How well did you know this?
1
Not at all
2
3
4
5
Perfectly
66
Q

Angiomyolipoma (AML)

A

An angiomyolipoma is a benign hamartoma made up of blood vessels (angio), smooth muscle (myo), and fat (lipoma).

Although benign, there is an increased risk of hemorrhage if >4 cm in size. The hemorrhage may be caused by microaneurysm rupture within the vascular elements of the AML.

On ultrasound, AML is echogenic due to the fat component. There is considerable overlap between the ultrasound appearance of AML and renal cell carcinoma. About one third of AML demonstrate shadowing, which is a specific finding for AML. Multiple AML are seen in tuberous sclerosis.

How well did you know this?
1
Not at all
2
3
4
5
Perfectly
67
Q

Oncocytoma

A

Oncocytoma is a benign renal tumor arising from tubular cells.

On ultrasound, oncocytoma is indistinguishable from renal cell carcinoma (RCC). It may be hypoechoic, isoechoic, or hyperechoic. A spoke-wheel vascular pattern is sometimes seen on color Doppler.

Due to imaging overlap with RCC, oncocytomas are treated surgically, even if the typical stellate or spoke-wheel vessels are seen.

How well did you know this?
1
Not at all
2
3
4
5
Perfectly
68
Q

Renal cell carcinoma (RCC)

A

Renal cell carcinoma (RCC) is the most common solid renal mass.

The staging of RCC uses the Robson system, which is discussed in the genitourinary section.

RCC is most often isoechoic to renal cortex, but can occasionally be hypoechoic or even hyperechoic (mimicking AML). A hypoechoic rim and intratumoral cystic changes are typically seen only in RCC, which may help distinguish it from AML.

In the presence of a renal mass, the renal veins must be carefully evaluated as RCC has a propensity for venous invasion. Venous invasion is Robson stage IIIA, and the presence of venous invasion has important implications for surgical approach.

Color and spectral Dopller are helpful in differentiating bland renal vein thrombus (which would not be stage IIIA) from tumor thrombus. Tumor thrombus will have color Doppler flow with an artrial waveform.

How well did you know this?
1
Not at all
2
3
4
5
Perfectly
69
Q

Renal lymphoma

A

Renal lymphoma (most commonly high-grade B-cell) may disseminate hematogenously or spread directly from the retroperitoneum to the kidney. Primary renal lymphoma is very rare and of uncertain origin as there is no native lymphoid tissue within the kidney.

The most common imaging presentation of renal lymphoma is multiple hypoechoic renal masses. Retroperitoneal adenopathy is usually present. A solitary mass is an uncommon presentation. Diffuse lymphomatous infiltration producing nephromegaly is rare.

How well did you know this?
1
Not at all
2
3
4
5
Perfectly
70
Q

Potential pitfalls in diagnosing a cystic lesion

A

Renal scanning should be performed with multiple angles of insonation to differentiate hydronephrosis from a renal sinus cyst (parapelvic or peripelvic cyst). In hydronephrosis, the dilated spaces will all connect.

Color Doppler should always be utilized, as a renal artery aneurysm may mimic a cyst in grayscale.

How well did you know this?
1
Not at all
2
3
4
5
Perfectly
71
Q

Simple cortical cyst

A

A simple renal cyst should have the sonographic hallmarks of a simple cyst, featuring an imperceptibly thin wall, anechoic internal contents, and posterior through transmission.

Harmonic imaging can be helpful in confirming the diagnosis of simple renal cyst by eliminating artifactual low-level internal echoes.

How well did you know this?
1
Not at all
2
3
4
5
Perfectly
72
Q

Renal sinus cyst

A

A cyst in the renal sinus may be a peripelvic or parapelvic cyst. Peripelvic cysts are secondary to lymphatic obstruction and are often multiple. In contrast, a parapelvic cyst is a renal parenchymal cyst that herniates into the renal sinus and is usually solitary.

When multiple renal sinus cysts are present (most commonly peripelvic cysts), the appearance may mimic hydronephrosis. In contrast to hydronephrosis, renal sinus cysts will not be contiguous with each other.

How well did you know this?
1
Not at all
2
3
4
5
Perfectly
73
Q

Renal abscess

A

Renal infection, discussed below, may appear as a complex cystic renal mass.

How well did you know this?
1
Not at all
2
3
4
5
Perfectly
74
Q

Cystic renal cell carcinoma

A

Although most cases of RCC present as a solid renal mass, a significant minority may present as a complex cystic lesion. Worrisome ultrasound findings of a complex cystic mass include thick septa, irregular wall thickening, and a mural nodule.

The Bosniak classification of complex renal masses is based on CT appearance and depends on the enhancement. The Bosniak classification is described in the genitourinary imaging section.

How well did you know this?
1
Not at all
2
3
4
5
Perfectly
75
Q

Acute diffuse pyelonephritis

A

Pyelonephritis is infection of the renal parenchyma, usually by gram-negative urinary tract organisms that ascend from the lower genitourinary tract.

The most common ultrasound appearance of pyelonephritis is a normal kidney. Occasionally generalized renal edema and engorgement can be seen.

How well did you know this?
1
Not at all
2
3
4
5
Perfectly
76
Q

Focal pyelonephritis

A

Focal pyelonephritis is a focal or multifocal infection of the renal parenchyma.

The classic ultrasound appearance is a hypoechoic mass (or masses) with low-amplitude echoes that disrupts the corticomedullary junction. A distinct wall is lacking.

How well did you know this?
1
Not at all
2
3
4
5
Perfectly
77
Q

Renal abscess

A

A renal abscess is a focal necrotic parenchymal infection with a defined wall. Urinalysis may be negative up to 30% of the time if the infection does not involved the collecting system.

Small abscesses (<3 cm) often undergo a trial of conservative medical therapy, while larger abscesses are typically drained.

Ultrasound shows a fluid-filled renal mass with a distinct wall, which may be multiloculated.

How well did you know this?
1
Not at all
2
3
4
5
Perfectly
78
Q

Emphysematous pyelonephritis

A

Emphyesematous pyelonephritis is a complication of acute pyelonephritis characterized by replacement of renal parenchyma by gas. It is caused by gas-forming organisms, most commonly E. coli. Emphysematous pyelonephritis is almost exclusively seen in diabetic or immunocompromised individuals.

Emphysematous pyelonephritis is a surgical emergency requiring broad-spectrum antibiotics and emergent nephrectomy. Mortality can reach 40%.

Ultrasound shows high-amplitude echoes in the renal parenchyma representing gas locules with posterior dirty acoustic shadowing.

How well did you know this?
1
Not at all
2
3
4
5
Perfectly
79
Q

Tuberculous pyelonephritis

A

Tuberculosis pyelonephritis, caused by hematogenous spread of M. tuberculosis, is characterized by focal cavitary renal lesions with calcification.

Putty kidney is an atrophic, calcified kidney seen in end-stage renal tuberculosis.

How well did you know this?
1
Not at all
2
3
4
5
Perfectly
80
Q

Xanthogranulomatous pyelonephritis

A

Xanthogranulomatous pyelonephritis results from repeated cyscles of chronic low-grade infection caused by an obstructing calculus that leads to fibrofatty replacement of renal parenchyma.

On ultrasound, the kidneys are enlarged with areas of mixed echogenicity. A central stone is nearly universally present, which may be staghorn in morphology.

How well did you know this?
1
Not at all
2
3
4
5
Perfectly
81
Q

Pyonephrosis

A

Pyonephrosis is infection of an obstructed collecting system and is a surgical emergency. Treatment is emergent relief of obstruction, either with percutaneous nephrostomy or ureteral stent.

Ultrasound features echoes within a dilated collecting system. A fluid level may be present.

How well did you know this?
1
Not at all
2
3
4
5
Perfectly
82
Q

HIV associated nephropathy

A

The HIV virus may directly infect the renal parenchyma to produce HIV nephropathy, most commonly resulting in focal segmental glomerulosclerosis (FSGS). HIV nephropathy clinically presents with nephritic renal failure.

The kidneys are characteristically echogenic. Enlarged echogenic kidneys are specific for HIV nephropathy, although the kidneys are enlarged only about 20% of the time.

How well did you know this?
1
Not at all
2
3
4
5
Perfectly
83
Q

Autosomal dominant polycystic kidney disease (ADPKD)

A

Autosomal dominant polycystic kidney disease (ADPKD) is the most common cause of multiple renal cysts in adults. ADPKD is associated with cysts in the liver and other visceral organs.

15% of patients have saccular cerebral aneurysms.

The natural history of ADPKD is renal failure by middle age.

ADPKD does not confer an increased risk of renal cell carcinoma; however, complex cysts with internal hemorrhage are difficult to distinguish from renal cell carcinoma.

Imaging of ADPKD shows markedly enlarged kidneys with innumerable cysts of varying size and echogenicity.

How well did you know this?
1
Not at all
2
3
4
5
Perfectly
84
Q

Autosomal recessive polycystic kidney disease (ARPKD)

A

Autosomal recessive polycystic kidney disease (ARPKD) is a diagnosis of infancy. Prognosis is poor. If the child survives infancy, hepatic fibrosis usually develops.

ARPKD presents in utero as enlarged echogenic kidneys since the cysts are too small too be individually resolved by ultrasound.

How well did you know this?
1
Not at all
2
3
4
5
Perfectly
85
Q

Acquired renal cystic disease

A

Patients on long-term dialysis often develop many small renal cysts superimposed upon atrophic kidneys. Acquired cystic disease does not confer an increased risk of renal cell carcinoma, in contrast to ADPKD.

How well did you know this?
1
Not at all
2
3
4
5
Perfectly
86
Q

Approach to renal transplant

A

The goal of ultrasound evaluation after renal transplant is to determine whetehr there is treatable surgical or vascular complication. Ultrasound is not useful for differentiating among the various kinds of parenchymal rejection.

The transplanted kidney is implanted in the right or left iliac fossa (right more commonly), and is often very well imaged due to its superficial location.

An elevated RI (>0.7) suggests renal dysfunction, but this finding is nonspecific.

How well did you know this?
1
Not at all
2
3
4
5
Perfectly
87
Q

Surgical complications following renal transplant

A

Ureteral obstruction is apparent on ultrasound as hydronephrosis.

Fluid collection (blood, pus, urine) is highly dependent on timing: Immediately postoperative: Hematoma; 1-2 weeks postoperative: Urinoma; 3-4 weeks postoperative: Abscess; 2nd month and beyond: Lymphocele.

How well did you know this?
1
Not at all
2
3
4
5
Perfectly
88
Q

Vascular complications following renal transplant

A

Renal vein thrombosis: The renal artery Doppler may show reversal of diastolic flow.

Renal artery stenosis: Elevated flow velocities are seen at the site of stenosis, with a parvus et tardus waveform distal to the stenosis. Usually takes several weeks to months to develop.

Pseudoaneurysm is usually due to renal biopsy.

How well did you know this?
1
Not at all
2
3
4
5
Perfectly
89
Q

Medical complications

A

Medical complications generally cannot be differentiated on ultrasound. Biopsy is necessary for diagnosis, although the time elapsed since the transplant may be a helpful clue.

Hyperacute rejection: Occurs in first few hours after transplant. Hyperacute rejection is very rare, and is due to ABO blood type incompatibility.

Acute tubular necrosis (ATN): Occurs in the immediate flow postoperative days. ATN is usually a sequela of pre-impantation ischemia.

Acute rejection: Occurs within three months of transplant.

Chronic rejection: Occurs after three months of transplant.

Drug toxicity may be caused by cyclosporine, which is nephrotoxic.

How well did you know this?
1
Not at all
2
3
4
5
Perfectly
90
Q

Post-transplant lymphoproliferative disorder (PTLD)

A

Post-transplant lymphoproliferative disorder (PTLD) is a type of lymphoma that is thought to be due to immune suppression and Epstein-Barr virus proliferation.

PTLD can arise anywhere in the body. Any new mass in any organ in a transplant patient should raise concern for potential PTLD.

Ultrasound of renal PTLD will show an amorphous hypoechoic mass that may simulate a fluid collection on grayscale images. Unlike fluid, PTLD will demonstrate Doppler flow.

How well did you know this?
1
Not at all
2
3
4
5
Perfectly
91
Q

Medullary nephrocalcinosis

A

Any cause of hypercalcemia and hypercalciuria can cause medullary calcification.

Hyperparathyroidism is the most common cause of medullary nephrocalcinosis.

Renal tubular acidosis (distal type).

Medullary sponge kidney is caused by ectatic tubules in the medullary pyramids leading to stasis and stone formation.

Papillary necrosis.

In a child, treatment with furosemide can lead to medullary nephrocalcinosis.

How well did you know this?
1
Not at all
2
3
4
5
Perfectly
92
Q

Cortical nephrocalcinosis

A

Much more rare than medullary nephrocalcinosis, cortical nephrocalcinosis is due to diffuse cortical injury.

Acute cortical necrosis.

Hyperoxaluria (rare).

Alport syndome.

Autosomal recessive polycystic kidney disease.

How well did you know this?
1
Not at all
2
3
4
5
Perfectly
93
Q

Echogenic kidneys

A

Echogenic kidneys are most commonly due to medical renal disease, such as diabetic nephropathy, glomerulosclerosis, acute tubular necrosis, etc.

HIV nephropathy causes enlarged and echogenic kidneys.

How well did you know this?
1
Not at all
2
3
4
5
Perfectly
94
Q

Echogenic renal mass

A

Angiomyolipoma (AML). A shadowing echogenic renal mass is relatively specific for AML.

Malignant neoplasm (atypical appearance).

Renal calculus.

Intrarenal gas.

Milk of calcium, caused by cystals precipitating out of supersaturated solution.

Sloughed papilla, secondary to papillary necrosis, may appear as an echogenic mass in the collecting system.

How well did you know this?
1
Not at all
2
3
4
5
Perfectly
95
Q

Scrotal Anatomy

A

Epididymis: The epididymis carries sperm away from the testical to the vas deferens. The epididymis is composed of head, body, and tail. The head may measure up to 10 mm. The epididymis is normally hypoechoic and has less blood flow compared to the testicle. Relatively increased epididymal blood flow can be seen in epididymitis.

Mediastinum testis: The mediastinum testis is fibrous tissue in the hilum of the testicle, from which fibrous septa radiate towards the testicular periphery. It provides structural support to the rete testis.

Rete testis: The rete testis is a network of tubules that carries sperm from the seminiferous tubules to teh vas deferens. It functions to concentrate sperm.

How well did you know this?
1
Not at all
2
3
4
5
Perfectly
96
Q

Approach to a testicular mass

A

Intratesticular masses are usually malignant (90-95%). Conversely, most extratesticular masses are benign in an adult, although a pediatric mass in this location may be malignant.

The retroperitoneum should always be evaluated if an intratesticular mass is seen. Likewise, if retroperitoneal adenopathy is seen in a reproductive-age male, the testicles should always be examined.

Most scrotal masses are hypoechoic relative to normal testicular parenchyma.

On Doppler, most masses will have increased vascularity with high diastolic flow, producing a low resistance waveform.

How well did you know this?
1
Not at all
2
3
4
5
Perfectly
97
Q

Malignant germ cell tumor (GCT): Seminoma

A

Seminoma is the most common testicular malignancy. It has a favorable prognosis. Seminoma typically occurs in middle-aged men. Uncommonly, hCG may be elevated.

The spermatocytic subtype of seminoma occurs in slightly older men (mid 50s) and has excellent prognosis with orchiectomy only. Tumor markers are not elevated.

How well did you know this?
1
Not at all
2
3
4
5
Perfectly
98
Q

Malignant germ cell tumors: Nonseminomatous germ cell tumors (NSGCT)

A

Nonseminomatous germ cell tumors (NSGCT) include embryonal carcinoma, teratoma, yolk sac tumor, choriocarcinoma, and mixed subtypes. Mixed germ cell tumor is the most common NSGCT, and is the second most commo primary testicular malignancy after seminoma. The most common components of mixed NSGCT are embryonal carcioma and teratoma. Embryonal cell carcinoma in its pure form is rare and in adults is typically seen as a component of mixed germ cell tumors. The infantile form, called endodermal sinus tumor or yolk sac tumor, the most common testicular tumor of infancy. AFP is elevated. Teratoma is rare in its pure form in adults, but is seen in 50% of mixed NSGCT. Teratoma is classified as mature, immature, and malignant. In adults, teratomas are usually malignant. In children, teratomas, are usually benign, with the mature subtype most commonly seen. Choriocarcinoma is the most aggressive and rare NSGCT. Choriocarcinoma metastasizes early, especially to brain and lung. Metastasis tend to be hemorrhagic. hCG is always elevated and gynecomastia may result from elevated chorionic gonadotropins.

NSGCT generally occur in younger pateitns compared to seminomas, typically in young men in their 20s and 30s. NSGCT tend to be more aggressive than seminomas. Local invasion into the tunica albuginea and visceral metastases are common.

A heterogenous testicular mass that contains solid and cystic components and coarse calcificiation is a typical appearance for a NSGCT. It is not possible to distinguish the various subtypes of NSGCT on sonography.

How well did you know this?
1
Not at all
2
3
4
5
Perfectly
99
Q

Burnt-out germ cell tumor

A

Burnt-out germ cell tumor is a primary testicular neoplasm that is no longer viable in the testicle even though there is often viable metastatic disease, especialy retroperitoneal.

In the testicle, focal calcification with shadowing is characteristic. A mass may or may not be present.

Treatment is orchiectomy in addition to systemic chemotherapy.

How well did you know this?
1
Not at all
2
3
4
5
Perfectly
100
Q

Testicular microlithiasis

A

Testicular microlithiasis is multiple punctate testicular calcifications.

There is controversial association between microlithiasis and testicular neoplasm. While the overall absolute risk for developing testicular cancer remains very small inthe presence of microlithiasis, the relative risk may be increased.

Current guidelines do not support screening by ultrasound or tumor markers, but patients with microlithiasis may perform self-examinations and be seen in follow-up as needed.

At least five microcalcifications must be present per image to be called microlithiasis. If there are fewer than five microcalcifications the term limited microlithiasis isused.

Microlithiasis can produce a starry sky appearance is calcifications are numerous. In the liver, hepatitis can cause a starry sky appearance due to increased echogenicityof the portal triads.

How well did you know this?
1
Not at all
2
3
4
5
Perfectly
101
Q

Testicular metastases

A

The most common metastases to the testicles are leukemia and lymphoma, as the relevant chemotherapeutic agents do not cross the blood-testis barrier.

Hematologic malignancies typically present in older patients, tend to be bilateral, and may be infiltrative with diffuse testicular enlargement.

How well did you know this?
1
Not at all
2
3
4
5
Perfectly
102
Q

Benign testicular tumors

A

An epidermoid is a keratin-filled cyst with a distinctive onion-ring appearance of concentric alternating rings of hypo- and hyperechogenicity. If suspected, local excision is performed instead of the standard orchiectomy typically performed for presumed malignant masses.

Sex-cord stromal tumors are 90% benign but are sonographically indistinguishable from malignant tumors. Orchiectomy is therefore the standard treatment. Leydig cell tumor can present with gynecomastia due to estrogen secretion. Sertoli cell tumor is associated with Peutz-Jeghers and Klinefelter syndromes.

How well did you know this?
1
Not at all
2
3
4
5
Perfectly
103
Q

Sarcoidosis

A

Sarcoidosis may involve either the testis, the epididymis, or both. Scrotal involvement is rare, but presents clinically as painless scrotal enlargement.

The ultrasound appearance of testicular sarcoid is indistinguishable from a solid malignant mass. If sarcoidosis is suggested by clinical history, the testicular mass must be biopsied to exclude malignancy. Without tissue pathology, a mass cannot be assumed to be sarcoid.

How well did you know this?
1
Not at all
2
3
4
5
Perfectly
104
Q

Benign testicular tumor mimics

A

Congenital adrenal rests are embryologic remnants of adrenal tissue trapped within the testis. These are typically seen in newborns with congenital adrenal hyperplasia. Adrenal rests appear as bilateral hypoechoic masses and classically enlarge with ACTH exposure.

Polyorchidism/supernumerary testis: An extra testicle has an identical imaging appearance to normal testicular parenchyma. Extranumerary testes carry a slightly increased risk of torsion and testicular cancer.

How well did you know this?
1
Not at all
2
3
4
5
Perfectly
105
Q

Extra-testicular masses

A

In contrast to intratesticular masses, extratesticular masses are usually benign. Up to 16% of extratesticular masses may be malignant, however, and ultrasound cannot reliably differentiate benign from malignant masses.

Spermatic cord lipoma is the most common extratesticular neoplasm ovverall.

Benign adenomatoid tumor of the tunica albuginea is the most common epididymal neoplasm.

How well did you know this?
1
Not at all
2
3
4
5
Perfectly
106
Q

Hydrocele

A

A hydrocele is excess fluid in the scrotum surrouding the testicle. Most are asymptomatic.

A hydrocele may be congenital (due to patent processus vaginalis in utero or infancy), idiopathic, or post-inflammatory. Regardless of etiology, there is never fluid at the bare are where the testicle is attached to the tunica vaginalis.

How well did you know this?
1
Not at all
2
3
4
5
Perfectly
107
Q

Hematocele

A

A hematocele is blood in the scrotum due to trauma or torsion.

How well did you know this?
1
Not at all
2
3
4
5
Perfectly
108
Q

Varicocele

A

A varicocele is a dilated venous pampiniform plexus in the scrotum. A primary varicocele is due to incompetent valves of the internal spermatic vein. A secondary varicocele is due to inceased venous pressure caused by an obstructing lesion.

Varicocele is a common cause of infertility, seen in up to 40% of males presenting to an infertility clinic.

Varicoceles are much more common on the left as the left testicular vein drains into the left renal vein and the superior messenteric artery can compress the left renal vein. In contrast, the right testicular vein drains directly into the infrarenal IVC.

85% of varicoceles are left-sided and 15% are bilateral. An isolated right-sided varcocele should prompt a search for a right-sided retroperitoneal mass.

On ultrasound, varicoceles appear as multiple tubular and serpentine anechoic structures >2 mm in diameter in the region of the upper pole of the testis and epididymal head. The varicoceles follow the spermatic cord into the inguinal canal and can be compressed by the transducer. Careful optimization of Doppler paremeters shows the slow venous flow within the varicocele.

How well did you know this?
1
Not at all
2
3
4
5
Perfectly
109
Q

Epididymal cysts and spermatocele

A

An epididymal cyst is an anechoic fluid-containing cyst that can occur anywhere in the epididymis.

A spermatocele is cystic dilation of the epididymis filled with spermatozoa, usually occuring in the epididymal head. Classic ultrasound appearance is an epididymal cyst with internal low-level mobile echoes.

A simple epididymal cyst and a spermatocele cannot always be reliably distinguished by ultrasound.

How well did you know this?
1
Not at all
2
3
4
5
Perfectly
110
Q

Simple testicular cyst

A

A simple testicular cyst meets sonographic criteria for a simple cyst (smooth posterior wall, imperceptible wall thickness, completely anechoic, posterior through transmission).

111
Q

Tubular ectasia of rete testis

A

Tubular ectasia of the rete testis is nonpalpable, asymptomatic, cystic dilation of the tubules at the mediastinum testes caused by epididymal cyst or spermatocele.

Tubular ectasia of the rete testis is common in older patients and may be bilateral.

Imaging shows numerous tiny dilated structures in teh region of the mediastinum testis, often seen in conjunction with an epididymal cyst/spermatocele.

Important to be aware of only as a tumor mimic. Tubular ectasia is benign and no treatment is necessary.

112
Q

Tunical cyst

A

The tunica albuginea is the capsule overlying the testis. A cyst of the tunica albuginea presents as a palpable superficial nodule that resembles a BB.

Sonography shows a typically small, simple, extra-testicular cyst.

No treatment necessary.

113
Q

Testicular torsion

A

Testicular torsion is twisting of the testicle around the spermatic cord and the vascular pedicle. Torsion presents with acute scrotal pain and is a surgical emergency.

Torsion may lead to irreversible testicular infarction if not de-torsed within a few hours. De-torsion within 6 hours has an excellent prgonosis. De-torsion after 24 hours has a poor prognosis for testicular salvage.

The bell-clapper deformity predisposes to torsion due ot a small testicular bare area. The bare area is the testicular attachment site and normally on the time elapsed since torsion: Hyperacute (within a few hours): Ultrasound shows a hyperechoic and shadowing torsion knot of twisted epididymis and spermatic cord, with not blood flow in the affected testicle. Acute (between a few hours and 24 hours): Affected testicle is enlarged and heterogenous. Missed torsion (>24 hours): Affected testicle is enlarged and mottled, with scrotal skin thickening and increased flow in the scrotal wall. A complex or septated hydrocele may be present.

114
Q

Segmental infarction

A

Segmental infarction is a focal testicular infarction that can be due to microvascular thrombosis from acute inflammation, vasculitis, or sickle cell disease.

Patients are typically in their 30s and present with acute pain which may mimic epididymitis or torsion clinically.

The typical appearance of infarction is a wedge-shaped hypoechoic area with no flow on Doppler.

The primary differential consideration of infarction is a hypovascular tumor. Infarcted tissue may undergo necrosis, making differentiation from tumor even more difficult. MRI may be helpful to distinguish infarction from tumor in ambiguous cases to potentially spare the patient form orchiectomy.

115
Q

Scrotal hematoma

A

The sonographic appearance of an acute scrotal hematoma is an echogenic, extratesticular mass with no Doppler flow. When large, the hematoma can compress the testicle.

When the hematoma evolves into a complex, multiseptated mass-like lesion, the distinction between the extratesticular hematoma and the testicle may become difficult. Proper distinction is necessary to avoid mistaking the hematoma for a testicular mass.

116
Q

Testicular contusion

A

Testicular contusion produces a peripheral, hypoechoid lesion that may mimic a tumor.

Even with a history of trauma, a suspicious testicular lesion requires further evaluation to exclude malignancy, typically with a short-term follow-up.

117
Q

Testicular rupture

A

Testicular rupture causes capsule disruption, often with protrusion of testicular parenchyma through the defect. Rupture is often associated with a testicular hematoma or contusion.

Prompt diagnosis is critical, as testicular viability is dependent upon timely repacking of the seminiferous tubules back inside the capsule.

Testicular rupture results in disruption of the blood-testis barrier and may be associated with future infertility due to the formation of anti-spermatozoa antibodies.

118
Q

Epididymitis

A

Epididymitis is infection of the epididymis, almost always ascending from the urinary tract.

The classic clinical presentation of epididymitis is acute unilateral scrotal pain.

The main differential based on clinical presentation is testicular torsion. In contrast to torsion, epididymitis features normal testicular blood flow.

A key ultrasound finding of epididymitis is an enlarged epididymis with increased Doppler flow relative to the testicle (normally, the epididymis has less Doppler flow than the testicle). An associated hydrocele may be present, which often contains low-level echoes.

119
Q

Epididymo-orchitis

A

Epididymo-orchitis is infection that has spread from the epididymis to the testicle.

Epididymo-orchitis has a similar ultrasound appearance to epidiymitis, but blood flow to the testicle will also be increased.

Infection and secondary inflammation can cause venous hypertension, which is a risk factor for focal testicular ischemia.

120
Q

Fournier gangrene

A

Fournier gangrene is necrotizing fasciitis of the scrotum and perineum, a highly morbid and surgically emergent condition.

Infection is usually polymicrobial.

The key imaging fidning is subcutaenous gas, which appears on ultrasound as multiple echogenic reflectors in the subcutanous tissues with dirty shadowing.

121
Q

Vascular ultrasound Angle correction

A

The Doppler signal is proportional to cos(ø). There is no Doppler shift at 90°.

All measurements of velocity should be made at a consistent angle (typically 60). Measurements shouldnever be taken at an angle greater than 60.

122
Q

Overview of peak systolic velocity

A

Peak systolic velocity (PSV) is usually the most accurate method to evaluate the degree of arterial stenosis. PSV is elevated proximal to and at the site of stenosis.

PSV may be decreased distal to a hemodynamically significant stenosis.

The differential diagnosis of increased PSV includes: Downstream (distal) stenosis; Compensatory flow, contralateral to an obstruction or severe stenosis; Physiologic hyperdynamic state in a healthy young patient.

The differential diagnosis of decreased PSV includes: Upstream (more proximal stenosis; Poor cardiac pump function; Near-total occlusion.

123
Q

Color and spectral Doppler parameters

A

By convention, for images to be obtained in the sagittal plane, the patien’ts head is on the left side of the image and the feet are on the right.

In general, ultrasound parameters are optimized so that arteries are red and normal arterial flow is above the baseline. Certain parameters need to be adjusted so that arteries above the heart (which are normally heading towards the head) appear similar to arteries below the heart (which normally are heading towards the feet): The color scale can be changed: Colors above the baseline go towards the probe. Spectral Doppler baseline inversion can be changed: Positive waveforms go towards the probe.

124
Q

Evaluation of the carotid arteries

A

There are three components to the carotid artery exam: Evaluation of plaque morphology, hemodynamic evaluation, and waveform analysis.

125
Q

Carotid plaque morphology

A

Plaque morphology is evaluated on grayscale imaging (without Doppler) and is described in terms of absolute percent stenosis.

<50% cross-sectional area plaque would not be expected to be hemodynamically significant.

>50% luminal plaque is expected to show elevation in peak systolic velocity.

126
Q

Carotid Hemodynamic evaluation of stenosis

A

Normal peak systolic velocity (PSV) in large arteries is 60-100 cm /sec.

PSV tends to be elevated at a site of significant stenosis. Per the Society of Radiologists in Ultrasound (SRU) criteria, established in 2003: >125 cm/sec suggests >50% stenosis; >230 cm/sec suggests >70% stenosis. Potential pitfall: An occluded or nearly occluded artery may have no detectable flow.

An elevated ratio of internal carotid artery to common carotid artery (ICA/CCA) PSV is a useful secondary sign of ICA stenosis. <2 is normal. >2 suggests >50% ICA stenosis. >4 suggests >70% ICA stenosis.

End diastolic velocity of >100 cm/sec suggests >70% stenosis.

In high and low flow states, the ICA/CCA ratio is more useful than the absolute PSV.

127
Q

Carotid waveform analysis

A

Stenosis downstream (distal) to transducer (outflow lesion): Spectral waveform is high resistance and high velocity in morphology, characterized by decreased diastolic flow. The systolic upstroke is normal and rapid. Spectral broadening and aliasing may be present. Spectral broadening describes the widened distribution of RBC velocities due to disruption of laminar flow. Aliasing is an artifact where the highest velocities ar eshown to have a reversed flow.

Stenosis upstream (proximal) to transducer (inflow lesion): Spectral waveform is low resistance and low velocity in morphology, with relatively increased diastolic flow. Systolic upstroke is slowed, producing the tardus et. parvus waveform.

128
Q

Carotid stenosis

A
129
Q

Renal artery stenosis: Criteria and protocol

A

A peak systolic velocity of >180 cm/sec is consistent with renal artery stenosis. Normal aortic and renal artery velocity is 60-100 cm/sec

A renal artery to aortic velocity ratio of >3.5 is also consistent with renal artery stenosis.

Reduced or absent diastolic flow is suggestive of a stenosis distal to the area of interest.

As with the carotid artery, a tardus et parvus waveform on spectral Doppler is suggestive of a stenosis proximal (upstream) to the transducer, known as an inflow lesion.

An elevated renal resistive index (>0.7) is nonspecific, but may indicate renal artery stenosis. The resistive index is calculated as follows: RI = (PSV-EDV)/PSV, where PSV is peak systolic velocity and EDV is end-diastolic velocity. RI is measure in teh segmental arteries of the upper, mid and lower poles. Elevated resistive indices can also be seen in acute urinary obstruction or medical renal disease.

130
Q

Atherosclerotic renal artery stenosis

A

Atherosclerosis is by far the most common cause of renal artery stenosis, typically affecting the ostium of the renal artery.

131
Q

Fibromuscular dysplasia (FMD)

A

Fibromuscular dysplasia (FMD) is a vasculitis that primarily affects the renal and carotid arteries in middle-aged females.

The most common location of stenosis in FMD is the distal two thirds of the renal artery.

The classic angiographic appearance of FMD is a string of pearls caused by multifocal alternating stenoses and post-stenotic dilations.

132
Q

Lower extremity venous system anatomy

A

The superficial venous system is composed of the great and small saphenous veins. The great saphenous veins drains into the common femoral vein. Although the great saphenous vein is technically part of the superficial system, clots near the saphenofemoral junction are typically treated with anticoagulation because of their propensity to become dislodged. The small saphenous vein drains into the popliteal vein (which continues proximally as the femoral vein). Clots in the small saphenous vein are typically not treated.

Deep venous system anatomy mirrors arterial anatomy: The common femoral vein (CFV) drains into the external iliac vein and begins at the level of the inguinal ligament. The CFV lies medial to the common femoral artery. CFV tributaries include the deep femoral and femoral veins. The femoral vein was previously called the superficial femoral vein. The term superficial femoral vein should be avoided as it wrongly implies that this vein is part of the superficial venous system. The three calf veins are the anterior tibial vein (lateral), peroneal vein (middle), and posterior tibial vein (medial), which join to form the popliteal vein (PV). The PV continues into the femoral vein.

133
Q

Overview of the deep venous thrombosis (DVT) examination

A

A lower extremity venous ultrasound exam should include venous compression, color and spectral Doppler, and evaluation of venous augmentation and respiratory variation.

Augmentation is the normal change in waveform when the calf is compressed. Lack of augmentation suggests a distal venous obstruction between the calf and the transducer.

Respiratory variation is the normal change in waveform when the patient inspires. Lack of respiratory variation suggests a proximal venous obstruction.

The popliteal, femoral, proximal deep femoral, and common femoral (including the saphenofemoral junction) veins should be imaged every 2-3 cm with and without compression.

134
Q

Venous compression

A

The hallmark sonographic finding of a DVT is a noncompressible vein with or without an intraluminal clot. A partially thrombosed vein may be partially compressible, while a completely thrombosed vein will not be compressible at all.

135
Q

Color Doppler

A

Color Doppler is almost always used to help localize the veins, but it is not necessary for diagnosing DVT.

Normal color Doppler flow in a noncompressible vein is suspicious for nonobstructing thrombus.

136
Q

Acute versus chronic deep venous thrombosis

A

While the diagnosis of DVT is usually straightforward, distinguishing between acute and chronic thrombus can be difficult. Evaluation of the clot’s echogenicity is not a reliable way to determine the acuity of the clot as artifactual echoes within the vein lumen can overlap with the clot.

Sonographic findings of chronic venous thrombus include clot retraction and poor visualization of the clot, only partial compressibility, irregularly echogenic and thickened vein walls, and prominent collateral veins.

137
Q

Abdominal aortic aneurysm

A

Ultrasound is a principle screening modality for abdominal aortic aneurysm, with a proven mortality benefit in 65-79-year-old-men who have ever smoked tobacco.

If an aneurysm is present, the diameter is measured in three orthogonal planes.

Aneurysms with an axial diameter of >5.5 cm should be considered for elective treatment.

Aneursysms 3-5.5 cm are typically followed.

138
Q

Aortic dissection

A

In aortic dissection, a tear in the intima allows blood into the media. The characteristic intimal dissection flap is typically echogenic.

Color Doppler may show flow in both true and false lumens, often with different flow rates.

139
Q

Hashimoto thyroiditis (chronic lymphocytic thyroiditis)

A

Hashimoto thyroiditis is an autoimmune disease that ultimately produces destruction of the thryoid gland parenchyma. It is the most common cause of hypothryoidism.

Hashimoto thyroiditis can present with a variety of clinical findings, thyroid function test results, and imaging appearances, dependent on the duration and severity of the disease.

Ultrasound may show either a diffusely nodular gland or a diffusely coarsened gland without a measurable nodule. The isthmus is characteristically thickened.

Patients with Hashimoto thyroiditis are at increased risk of thyroid lymphoma. Any rapidly growing nodule should raise suspicioun for lymphoma.

140
Q

Graves disease

A

Graves disease causes autoimmune activation of the TSH receptor, stimulating thyroid hormone synthesis and secretion. Patients clinically present with thyrotoxicosis.

The typical grayscale sonographic appearance of Graves disease is diffuse enlargement of the gland with a coarsened echotexture. The borders of the gland are often lobulated.

The key color Doppler finding is the thyroid inferno sign, which represents marked hypervascularity caused by arteriovenous shunting and enlarged peripheral vessels.

141
Q

Subacute thyroiditis (de Quervain thyroiditis)

A

Subacute (de Quervain) thyroiditis is granulomatous inflammation of the thyroid, thought to be viral in origin. The gland is usually tender and adjacent cervical adenopathy is common.

Ultrasound findings are non-specific and may feature a heterogenous gland with patchy areas of decreased echogenicity.

Subacute thyroiditis is treated with steroids. Follow-up ultrasound appearance can show a dramatic response to treatment.

142
Q

Multinodular gland

A

The term multinodular gland is preferred over multinodular goiter because goiter is a generic term for an enlarged gland, which can have numreous causes.

On imaging, a multinodular gland will appear enlarged with innumerable mixed cystic and solid nodules.

143
Q

Thyroid cancer (malignant thyroid nodule)

A

A typical ultrasound appearance of a nodule suspicious for malignancy is a solid lesion with punctate calcifications and irregular margins. A completely solid nodule is most suspicious. In general, there is a decreasing likelihood of cancer with increasing cystic components. In general, the likelihood of cancer is dependent on the pattern of calcification. Punctate calcifications are the most suspicious, followed by coarse or rim calcifications. Nodules without any calcifications have the least risk of being malignant. Taller-than-wide orientation is an ultrasound feature associated with thyroid cancer (analogous to the suspicious breast ultrasound finding of taller-than-wide orientation).

Papillary cancer is by far the most common histologic subtype of thyroid cancer, and confers teh best prognosis.

Follicular and medullary subtypes are less common and more aggressive. The anaplastic subtype is very rare and has the worst prognosis.

Thyroid lymphoma can be seen in patients with long-standing Hashimoto thyroiditis.

144
Q

Malignant adenopathy

A

Malignant lymph nodes often appear rounder in morphology than benign lymph nodes, with irregular margins and speckled or central calcifications.

Metastatic adenopathy from papillary thyroid cancer has a tendency to undergo cystic degeneration. In some cases (especially in young women), a cystic lymph node may be th only presenting feature of thyroid cancer and the thyroid gland may be completely normal by ultrasound.

145
Q

Normal parathyroid glands

A

The parathyroid glands are normally not visibile on ultrasound unless enlarged (due to parathyoid adenoma or hyperplasia).

The inferior parathyroids are located posterior to the inferior tip of the thyroid.

The superior parathyroids are located at the posterior aspect of the mid-thyroid.

146
Q

Parathyroid hyperplasia

A

In parathryoid hyperplasia, all four parathryoid glands are enlarged and usually visible on ultrasound.

147
Q

Parathyroid adenoma

A

A parathryoid adenoma represents a single overactive parathyroid.

A nuclear medicine Tc-99m sestamibi scan localizes the parathryoid tissue if an adenoma cannot be seen by ultrasound.

148
Q

Space of Retzius

A

The space of Retzius is an extraperitoneal potential space between the pubic symphysis and the bladder.

A mass in the space of Retzius (such as a hematoma) can displace the bladder posteriorly.

In contrast, pelvic or abdominal masses will displace the bladder inferiorly or anteriorly.

149
Q

Cervix

A

The cervix is seen transvaginally in the sagittal plane as the most proximal portion of the uterus directly posterior to the angle of the bladder.

The cervix is attached to the posterior edge of the bladder by the parametrium.

The cervix and uterus normally form a 90-degree angle.

Nabothian cysts are normal retention cysts due to occlusion of cervical glands.

150
Q

True and false pelves

A

The linea terminalis is a bony landmark separating the true (inferior) pelvis from the false (superior) pelvis. The linea terminalis is a composite of arcuate line of the ilium, the iliopectineal line, and the pubic crest.

Normally, the uterus and ovaries are in the true pelvis.

The dome of a full bladder extends into the false pelvis, pushing small bowel out of the true pelvis. The bladder acts as a sonographic window into the true pelvis.

151
Q

Normal variant uterine positions

A

About 20 degrees of uterine anteflexion is normal. As the bladder fills, the degree of anteflexion decreases.

Retroversion of the uterus may cause poor visualization of the fundus transabdominally.

Retroflexion of the uterus may cause even more severe sound attenuation of the uterine fundus.

152
Q

Scanning Orientation Pelvis

A

The sagittal scan plane is rotated 90 degrees between transabdominal and endovaginal orientation. The patient typically empties her bladder prior to endovaginal scanning.

153
Q

Overview of uterine malformation

A

Uterine malformations are due to abnormal development of the paired Mullerian ducts, which normally fuse during embryogenesis.

Complete failure of fusion -> didephlys uterus

Partial failure of fusion -> bicornuate uterus

Failure of resorption of inner-Mullerian septa -> septate uterus (by far most common uterine anomaly).

Congenital uterine abnormalities may be associated with urinary tract abnormalities such as renal ectopica or agenesis. The kidneys should be evaluated if a uterine malformation is seen.

Uterine anomalies increase the risk of reproductive problems since the uterine cavity (or cavities) are abnormally small and/or abnormal in contour.

The American Fertility Society (now known as the American Society of Reproductive Medicine) classifies Mullerian duct anomalies. Class I is uterine agensesis/hypoplasia, class II is a unicornuate uterus, and class III through VII represent the anomalies discussed below.

154
Q

Didephlys uterus (calss III)

A

A didephys uterus is two completely separate uteri and cervices, with complete endometrium, myometrium, and serosal surfaces on each side. 75% have a vaginal septum.

155
Q

Bicornuate uterus (class IV)

A

A bicornuate uterus has two uterine fundi, with a shared proximal lower uterine segment.

A bicornuate uterus may be bicornis bicollis (two cervices) or bicornis unicollis (one cervix).

156
Q

Septate uterus (class V)

A

A septate uterus consists of two uterine cavities, divided by a fibrous or muscular septum.

Septate uterus is the most likely of all uterine anomlies to be implicated in pregnancy loss since the fibrous septal tissue or myometrium is relatively avascular.

157
Q

Arcuate uterus (class VI)

A

An arcuate uterus is a small inpouching or concave surface of the fundus, which is considered a normal variant rather than an anomaly.

158
Q

Diethylstilbestrol (DES) usterus (class VII)

A

In utero exposure to diethylstilbestrol (DES) causes the fetus to develop a hypoplastic uterus with a T-shaped endometrial countour and is associated with an increased risk of clear cell vaginal cancer. DES hasn’t been used sicne the 1970s.

159
Q

Measuing the endometrium

A

The thickest portion of the endometrium should be measured transvaginally in the sagittal orientation. Ideally the endometrium should be measured in the menstrual phase.

Endometrial fluid is not included in the measurement: If endometrial fluid is present, the flanking endometrium is measured and the two components are summed.

160
Q

Cyclical endometrial thickness

A

Days 1-4: Menstrual phase. Endometrial thickness <4 mm. The endometrium is a thin, echogenic stripe in the menstrual phase.

Days 5-9: Early proliferative phase. Endometrial thickness 4-8 mm.

Days 10-14: Late proliferative (periovulatory) phase. Endometrial thickness 6-10 mm. Estrogen effects dominate in the proliferative phase, causing increased functional zone thickness. The endometrium becomes trilaminar with a hypoechoic zone between the endometrial cavity and the peripheral echogenic endometrium.

Days 15-28: Secretory phase: Endometrial thickness 7-14 mm. Progesterone effects dominate in the secretory phase, causing the functional layer to become even thicker, soft, and edematousas the spiral arteries become tortuous. The functional layer increases in echogenicity and becomes isoechoic relative to the basal layer. The endometrium reaches its maximum thickness and echogenicity in the late secretory phase.

161
Q

Endometrial polyp

A

An endometrial polyp can cause mucous discharge or irregular vaginal bleeding between cycles. Most endometrial polyps are benign, but larger polyps (>1.5 cm) or polyps occuring in postmenopasual patients may have malignant potential.

Ultrasound shows a focal nodular area of endometrial thickening, often with a feeding vessel by Doppler. A polyp is more definitively diagnosed by sonohysterogram, where saline is instilled into the uterus prior to transvaginal ultrasound.

162
Q

Tamoxifen effect

A

Tamoxifen is an estrogen agonist/antagonist used in the treatment of breast cancer. It acts as an antagonist at the breast and an agonist at the endometrium.

Tamoxifen can cause endometrial hyperplasia, metaplasia, and carcinoma.

Ultrasound shows irregular, cystic endometrium, which may simulate endometrial cancer or endometrial cystic atrophy.

Most women on tamoxifen are screened by ultrasound every 6 months for endometrial carcinoma.

163
Q

Endometrial cancer and postmenopausal endometrial thickness

A

Over 95% of endometrial carcinoma presents with postmenopausal bleeding. The main risk factor for endometrial cancer is prolonged estrogen exposure, which occurs with nulliparity, obesity, late menopause, and tamoxifen.

If the patient is not bleeding, the postmenopausal endometrium whould be <8 mm thick. Although an incidentally thickened endometrium may be a normal hyperplastic response to estrogen exposure, a thickened endometrium should always be regarded with suspicion for malignancy in a postmenopausal woman. If the endometrium is thicker than 8 mm in a postmenopausal woman, the patient should be evaluated further, typically via endometrial biopsy with or without hysteroscopy. Although uncommonly seen in the absence of bleeding, the finding most suggestive of endometrial carcinoma is the presence of ill-defined margins separating the endometrium and the myometrium.

If the patient is bleeding and the endometrium is less than 5 mm, the bleeding is caused by endometrial atrophy. There is negligible risk of endometrial cancer if the thickness is less than 5 mm.

Postmenopausal bleeding with an endometrium thicker than 5 mm may represent endometrial carcinoma and further workup is necessary. Note that the average endometrial thickness with endometrial carcinoma is 21 mm.

164
Q

Endometriosis

A

Endometriosis is ectopic endometrial tissue outside of the endometrial cavit.

An endometrioma is a hemorrhagic focus of ectopic endometrial tissue.

The classic ultrasound appearance of an endometrioma is well-defined complex cyst with homogenous low-level internal echoes and icnreased through transmission. Small linear echogenic foci are often seen at the cyst periphery. This classic appearance isn’t always seen and occasionally an endometrioma may appear similar to a neoplasm.

While the ovary is the most common site of involvement, endometriosis may affect the adnexa, pelvic viscera, or even organs outside of the pelvis, such as the brain.

165
Q

Adenomyosis

A

Adenomyosis is endometrial tissue within the myometrium. Adenomyosis typically presents with menorrhagia and pain.

Ultrasound shows heterogenous myometrium, typically more prominent in the posterior wall, associated with subendoetrial cysts. The uterus may be globular and enlarged and there if often poor differentiation of the endometrial-myometrial border. Focal adenomyosis, known as an adenoma, may simulate a fibroid.

166
Q

Fibroid (leiomyoma)

A

Fibroids are extremely common benign tumors of smooth muscle seen in 25% of white women and 50% of black women over age 30.

The typical ultrasound appearanc of a fibroid is a slightly heterogenous, hypoechoic uterine mass with linear bands of shadowing. Calcification is often seen. May undergo cystic degeneration and appear as an anechoic mass with posterior through transmission.

Fibroid location: Intramural: Location in the myometrium is the most common fibroid location. Submucosal: A submucosal fibroid may bulge into the endometrial canal, producing pain and bleeding. A submucosal fibroid can be resected hysteroscopically if >50% of the fibroid is intraluminal. An intracavitary fibroid is a variant of a submucosal fibroid located nearly entirely within the uterine cavity. Subserosal: A subserosal fibroid may simulate an adnexal mass if pedunculated, but Doppler will show blood supply coming from the uterus. Cervical: Rare, may simulate cervical cancer.

A lipoleiomyoma is a variant of fibroid that contains fat and is echogenic.

167
Q

Leiomyosarcoma

A

Malignant transformation of a fibroid to leiomyosarcoma is extremely rare.

A “funny looking fibroid” is much more likely to be a benign, inhomogenous fibroid rather than a leiomyosarcoma.

Tamoxifen increases the risk of leiomyosarcoma in addition to endometrial carcinoma.

168
Q

Endometrial fluid

A

It is never normal to have more than a tiny amount of fluid in the endometrial canal.

In a premenopausal woman, endometrial fluid can be due to bleeding from menses or spontaneous abortion.

In a postmenopausal woman, endometrial fluid can be due to cervical stenosis, and a careful evaluation for cervical malignancy should be performed.

169
Q

Uterine infections

A

Endometritis is inflammation or infection of the endometrium, and is commonly seen postpartum, typically with no specific findings on ultrasound. Gas in the uterus may be normal up to 3 weeks postpartum (seen in 7% of normal cases), but gas in the uterus later than 3 weeks after delivery may represent endometritis.

Pyometra (pus within the uterus) is very rare and usually due to outflow obstruction. An evaluation for cervical malignancy should be performed.

170
Q

Intrauterine device (IUD)

A

The ultrasound appearance of an intrauterine device (IUD) is dependent on the type of IUD: Mirena IUD (delivers progesterone): Shadowing structure in the endometrial canal. Conventional IUD: Highly echogenic.

Potential complications of an IUD are rare but serious: Increased risk of infection with prolonged IUD use, especially actinomycosis. When pregnancy occurs in the presence of an IUD, there is increased risk for ectopic pregnancy. Uterine perforation is very rare.

171
Q

Uterine arteriovenous malformation (AVM)

A

Uterine arteriovenous malformation may be congenital (very rare) or acquired iatrogenically (e.g., from a D&C)

Grayscale and color Doppler appearance shows an enlarged, heterogenous, and multicystic uterus. The appearance is similar to gestational trophoblastic disease (discussed in the first trimester of pregnancy section), but with a negative B-hCG.

172
Q

Post-Cesarean section complications

A

Bladder-flap hematoma is a rare complication of a low-transverse Cesarean, where a postsurgical hematoma forms in the vesicouterine space (posterior to the bladder, between the bladder and the uterus). Ultrasound of a bladder-flap hematoma will show a complex mass posterior to the bladder.

Subfasical hematoma is also a rare complication of Cesarean section due to extraperitoneal hemorrhage within the prevesical space (anterior to the bladder). Ultrasound shows a complex mass anterior to the bladder.

It is important to distinguish a subfascial hematoma from a bladder-flap hematoma as the surgical approach for repair is different.

173
Q

Ovaries anatomy and physiology

A

There is a dual blood supply to the ovary: The ovarian artery comes directly off the aorta to supply the lateral aspect of the ovary. A branch of the uterine artery arises from the internal iliac artery to supply the medial aspect of the ovary.

The fallopian tube is divided into four segments, from proximal to distal: Interstitial (intramural) is the narrowest segment. Isthmus. Ampulla. Infundibulum.

174
Q

Cyclical changes in the ovaries

A

Day 5-7 of the menstrual cycle: Multiple follicles become apparent in the ovary.

Day 8-13: One (or more) dominant follicles arise. 4-5 days before ovulation, the dominant follicle grows at the rate of approximately 2-3 mm/day. The maximal diameter of the dominant follicle is approximately 2 cm. The day prior to ovulation, a hypoechoic ring forms around the dominant follicle, which represents the granulosa layer separating from the theca.

Day 14: Ovulation. Physiologic bleeding occurs into the follicle at th time of ovulation, at which pointthe follicle is called the corpus hemorrhagicum. After ovulation, the corpus hemorrhagicum becomes the corpus luteum.

Days 15-20: The corpus luteum retains fluid over the next 4-5 days to reach a maximal size of approximately 3 cm.

Day 20-28. If pregnancy doesn’t occur, the corpus luteum involutes to become the corpus albicans, which cannot be seen by ultrasound.

If pregnancy does occur, the copus luteum develops into a gland secreting hCG. A prominent corpus luteum may be mistaken for an ectopic pregnancy due to its similar appearance. However, an ectopic pregnancy will only very rarely be in the ovary.

175
Q

Physiologic simple cyst (in a premenopausal patient)

A

A simple ovarian cyst is a round or oval anechoic structure with smooth and imperceptibly thin walls, posterior acoustic enhancement, and lack of worrisome features such as solid components, septations, or internal flow on color Doppler.

A simple ovarian cyst is a follicle that physiologically enlarges from estrogen stimulation as a normal part of the menstrual cycle.

The Society of Radiologists in Ultrasound (SRU) published a consensus in 2010 regarding management of asymptomatic ovarian and adnexal cysts imaged at ultrasound.

Cysts < 3 cm do not need to be described in the report, and there is no need for follow-up.

Cysts >3 and <5 cm should be mentioned in the report and described as benign, with no follow-up necessary.

Cysts >5 and <7 cm are almost certainly benign but should be followed annually.

Cysts >7 cm should be evaluated by MRI or surgery, as a full ultrasound assessment is difficult.

176
Q

Postmenopausal Simple ovarian cyst

A

Cysts <1 cm do not need to be reported or followed.

Cysts >1 cm and <7 cm are almost certainly benign, but should be described and followed annually with ultrasound (similar to premenopausal cysts >5 cm and <7 cm).

Cysts >7 cm should be evaluated by MRI or surgery, as a full ultrasound assessment is difficult (similar to a premenopausal cyst of the same size).

177
Q

Functional cyst

A

A functional cyst is the result of a follicular cyst that did not execute normally. Functional cysts include follicular cysts, corpus luteal cysts, and theca-lutein cysts.

A follicular cyst is a simple cyst larger than 25 mm, representing a follicle that did not undergo evaluation.

A corpus luteal cyst may grow to greater than 3 cm if it fails to involute normally. A corpus luteal cyst can have a variable appearance, but will often look like a complex ovarian cyst. High diastolic flow is often present, which can also be seen in ovarain cancer.

Theca-lutein cysts are often multiple and arise from elevated hCG. They can be seen in molar pregnancy, multiple gestations, or infertility patients on gonadotropins or clomiphene.

A hemorrhagic cyst is the result of hemorrhage into a functional cyst, most commonly a corpus luteum. Ultrasound findings can be suggestive, although a complex cyst should be followed-up at least once to ensure resolution. An acutely hemorrhagic cyst may be hyperechoic and potentially mimic a solid mass, but will usually show posterior enhancement. As the clot dissolves, the internal echo pattern becomes more complex to produce characteristic web-like internal echoes. Retractile mural clot featrures concave margins and absent Doppler flow. In contrast, a solid murla nodule features a convex margin and internal flow.

178
Q

Ovarian hyperstimulation syndrome (OHSS)

A

Ovarian hyperstimulation syndrome (OHSS) is a complication of fertility treatment, thought to be due to VEGF dysregulation causing capillary leak.

The criteria for diagnosis of OHSS include abdominal pain, enlargement of the ovary to greater than 5 cm, and presence of either ascites or hydrothorax. At least one additional laboratory or clinical symptom must be met, including elevated hematocrit (>45%), elevated WBC (>15,000), elevated LFTs, acute renal failure, or dyspnea.

OHSS increases the risk of ovarian torsion and ectopic pregnancy.

179
Q

Polycystic ovarian syndrome (PCOS)

A

Polycystic ovarian syndrome is a clinical syndrome of obesity, insulin resistance, anovulation, and hirsutism secondary to excess androgens.

Ultrasound criteria include >20 small follicles (most often arranged around the periphery of the ovary), none greater than 9 mm in diameter, and an ovarian volume >10 mL. Ovarian volume is calculated by multiplying the diameter of three orthogonal planes by 0.52.

The ovarian stroma is typically very vascular when evaluated by color Doppler.

A differential consideration is normal ovaries under the influence of oral contraceptives, although contraceptives will not increase the vascularity of the ovary.

180
Q

Paraovarian cyst

A

A paraovarian cyst is a simple cyst separate from the ovary, thought to be developmental in origin.

Paraovarian cysts are considered normal if <5 cm.

The main differential is an ovarian cyst. Ovarian cysts should be reported (and followed) if they are greater than 3 cm, while paraovarian cysts do not need to be followed unless they are greater than 5 cm.

181
Q

Peritoneal Inclusion Cyst

A

A peritoneal inclusion cyst is a septated fluid collection formed by adhesions from prior surgery. The ovary is always closely associated with the peritoneal inclusion cyst, either trapped within or adjacent to it.

It is important not to recommend surgery for treatment of a peritoneal inclusion cyst, as furgery surgery may create additional adhesions.

The main differential of a peritoneal inclusion cyst is a cystadenoma, which has thick septations and tends to exert mass effect.

182
Q

Dilated fallopian tube

A

The fallopian tube may become distended due to infection, inflammation, or traction from pelvic adhesions.

A hydrosalpinx is a fluid-filled fallopian tube lacking internal echoes. Ultrasound shows a dilated, anechoic, paraovarian tubular structure with incomplete septations. The incomplete septations represent infoldings of the tubular walls.

Hematosalpinx is a blood-filled fallopian tube that can be seen in the setting of a ruptured ectopic pregnancy or endometriosis. Imaging will show internal echoes within the dilated tube.

Pyosalpinx is a pus-filled fallopian tube resulting from pelvic inflammatory disease. As in hematosalpinx, imaging will show internal echoes within the dilated tube.

183
Q

Adnexal torsion

A

Adnexal torsion results from twisting of the ovarian vascular pedicle. This results in pain and potential vascular compromise to the ovary.

Acute pain is usually localized to the affected side. Pain may be episodic, especialy if the torsion is intermittent. Torsion occurs mainly in reproductive-age women, and commonly occurs in pregnancy. Torsion occurs more commonly on the right side due to the position of the sigmoid colon, which inhibits free rotation of the left adnexa. Torsion may clinically mimic appendicitis.

The ovary may be predisposed to torsion by a lead-point mass, most commonly a dermoid.

Because of the dual blood supply to the ovary (lateral from the ovarian vessels off the aorta, and medial from the uterine vessels from teh internal iliac), flow may still be detectable by color Doppler even with torsion.

The classic ultrasound presentation of torsion in a patient with acute pelvic pain is an enlarged ovary with free fluid and abnormal ovarian Doppler. The vascular pedicle may be twisted, which is very specific when seen. However, in the real world, the imaging findings tend to be less specific and may include: Enlarged ovary >4 cm in diameter. Unusual position of the affected ovary, which may even be found on the contralateral side. Follicles pushed to the periphery of the ovary. Free fluid in the pelvis. Variable Doppler findings: Complete lack of flow is concerning, although this is rarely seen. Other Doppler findings include intermittent flow, venous flow on spectral imaging, and even normal flow.

184
Q

Dermoid cyst

A

Dermoid cyst, also called a mature cystic teratoma, is the most common ovarian neoplasm. Technically, a tertoma contains all three primitive germ cell layers, while a dermoid cyst may contain only two. In general use, however, these terms are interchangeable.

Dermoid cysts are benign. Malignant transformation is very rare and typically occur in postmenopausal patients.

A dermoid cyst can act as a lead point for adnexal torsion.

The classic ultrasound appearance of a dermoid cyst is a complex ovarian cyst with an echogenic Rokitansky nodule, a mural nodule containing solid elements. The imaging appearance can be variable, however, and other common imaging features include: The dot-dash pattern describes interrupted echogenic lines thought to be produced by keratin fibers. The tip of the iceberg sign describes obscuration of the deeper contents due to high-attenuation material.

CTor MRI will confirm the presence of fat in ambiguous cases.

185
Q

Ovarian Cancer

A

Ovarian cancer is the sixth most common cancer in females, but is the leading cause of death from gynecologic malignancy as it commonly presents at an advanced stage.

Ultrasound findings suggestive of a malignant mass include: Mural nodule. Thick or irregular walls or septae. Solid components. High flow on color Doppler. Presence of ascites. Papillary projections.

The four histologic types of ovarian neoplasm are: Epithelial neoplasm (comprises two thirds of all ovarian neoplasms), germ cell tumor, sex cord-stromal tumor, and metastasis. The three subtypes of epithelial neoplasm are serous, mucinous, and endometroid. Endometroid carcinoma may arise from an endometrioma. Teratoma (dermoid cyst), is a germ cell tumor. Struma ovarri is a subtype of teratoma that is composed of mature, functioning thyroid tissue. Sex cord-stromal tumors include fibroma, thecoma, and fibrothecoma. Meigs syndrome is the triad of bening ovarian fibroma, ascites, and right pleural effusion. Tumors containing thecal cells produce estrogen and may cause endometrial carcinoma. Metastasis to the ovary is usually from an extra-pelvic primary. Common extra-pelvic primary cancers that may metastasize to the ovary include gastric and breast cancer. A Krukenberg tumor is an ovarian metastasis of a mucin-producing tumor, typically gastric or colonic. Endometrial cancer may also metastasize to the ovaries.

186
Q

Gestational Sac

A

The gestational sac is the earliest imaging finding in early pregnancy.

The intradecidual sign and the double decidual sign are two findings that may aid in the detection of very early pregnancy. The intradecidual sign and the double decidual sign are two findings that may aid in the detection of very early pregnancy. The intradecidual sign represents the gestational sac wihtin the thickened decidua, seen at <5 weeks. The double decidual sac sign represents the two echogenic rings encircling the gestational sac. It is most useful when seen, where it confirms the presence of an intrauterine pregnancy (IUP). The absence of a double decidual sign is considered indeterminate and may suggest either an IUP or the pseduogestational sac of an ectopic pregnancy. Practically, these signs are of limited clinical utility. With a positive pregnancy test and normal adnexae, any fluid collection in the uterus is overwhelmingly likely to represent a very early intrauterine pregnancy (IUP), regardless of the presence of the intradecidual, double decidual sac, or pseudogestational sac signs.

A gestational sac should be seen by transvaginal ultrasound if the B-hCG is greater than 1,500. The gestational sac is normally seen by 5 weeks.

The mean sac diameter (MSD) is the average diameter of the gestational sac measure in three orthogonal planes. The MSD is not routinely measured. If the MSD measures 8 mm, a yolk sac should be visible. If a yolk sac is not present, the pregnancy is unlikely to be successful. If the MSD measures 16 mm, an embryo should be visible. If an embryo is not seen when the MSD is 16 mm or greater, the pregnancy is unlikely to be successful.

A subchorionic hematoma is a potential complication of early pregancny caused by bleeding of the chorionic attachment. A small subchorionic hematoma surrouding the gestational sac is of no clinical significance. A large subchorionic hemoatoma will cause an approximately 40% chance of pregnancy failure.

187
Q

Yolk sac

A

Unlike in a chicken’s egg, the fetal yolk sac doesn’t contain any nutrients. It is a vestigial structure that functions in the early circulation before the development of the heart.

The yolk sac is normally seen by 5.5 weeks.

If the yolk sac is abnormally large (>6 mm), the pregnancy has a high chance of failure.

188
Q

Heartbeat and heart rate

A

A heartbeat is almost always detected when the embryo is large enough to be seen. It is unusual to see an embryo with a measurable crown rump length (CRL) without a heartbeat. If no heartbeat is seen in a visible embryo, the pregnancy has a high risk of failure.

It has long been accepted that a heartbeat should always be seen with a CRL of 5 mm, and lack of heartbeat was felt to be 100% diagnostic of a failed pregnancy.

Recent literature, however, suggests that definitive diagnosis of pregnancy failure based on absent heartbeat be witheld until the embryo has reached a size of 7 mm. Based on this, it may be prudent to recommend follow-up if no heartbeat is seen in an embryo under 7 mm, although the chance of a successful pregnancy in such a case is low.

Absence of a heartbeat by a gestational age of 6.5 weeks or greater is 100% diagnostic of a failed pregnancy. Note that it is only possible to be certain of pregnancy dating of the patient has had a previous ultrasound to establish early dating, or if the patient underwent IVF with a known transfer date. The date of the last menstrual perior is not reliable enough.

If the early heart beat is less than 90 bpm, there is very little chance that the pregnancy will be successful. There is no such thing as a “too fast” heart rate. In fact, embryos with a faster rate have had the highest chance of normal outcome. If the CRL is < 4 mm, <90 bpm is considered slow and >100 is normal. If the CRL is 5-9 mm, <110 bpm is considered slow and >120 is normal.

Heart rate is measured using M-mode Doppler.

189
Q

Dating convention

A

Early human civilizations may have associated sex with pregnancy, but not until recently has ovulation (occurring approximately 14 days prior to the first day of menses) been associated with conception.

The modern convention for dating pregnancy is a holdover from ancient times. Gestational age is calculated from the first day of the last menstrual cycle, not from conception. Therefore, a “6 week” pregnancy has really only been growing for 4 weeks. For IVF patients with a precisely known implantation date, 2 weeks are added to be consistent with the dating of spontaneous pregnancies.

190
Q

Assigning gestational age

A

Between 5 and 6 weeks getation, gestational age is determined based on three typical appearances of the early pregnancy.

Gestational sac only (with or without double sac sign): 5.0 weeks.

Gestational sac with a yolk sac, but without an embryo: 5.5 weeks.

Gestational sac with an embryo <3 mm and heartbeat: 6 weeks.

For embyros >3 mm in length, the crown rump length is used to assign gestational age using established reference tables. The CRL can estimate gestational age up to 12 weeks. After 12 weeks, dating is estimated using multiple fetal measurements.

191
Q

NEJM Fetal Viability

A
192
Q

Overview of ectopic pregnancy

A

Ectopic pregnancy is defined as a pregnancy outside of the endometrial cavity. Hemorrhage and resultant hypovolemic shock may be life-threatening to the mother.

The classic clinical presentation of ectopic pregnancy is a positive pregnancy test, vaginal bleeding, pelvic pain, and tender adnexal mass. This presentation is seen in less than 50% of patients.

193
Q

Ectopic Risk stratification

A

Any woman with a newly positive pregnancy test and either pain or bleeding is classified as a rule-out ectopic or clinically suspected ectopic patient and has about a 15% chance of having an ectopic pregnancy before any imaging is performed.

A rule-out ectopic patient may have an intrauterine pregnancy (IUP), ectopic pregnancy, or spontaneous abortion. The IUP may be normal, abnormal, or too early to detect.

The role of imaging is to determine if an IUP is present and to evaluate the common locations (especially the adnexae) where an ectopic may potentially be found.

194
Q

Definite diagnoses are the exception (ectopic)

A

More often than not, it is not possible to definitively diagnose ectopic pregnancy, except in the following situations: An extrauterine gestational sac that contains either an embryo or a yolk sac is 100% diagnostic of an ectopic pregnancy. A normal IUP, seen in conjunction with normal adnexae, is 100% diagnostic of a normal IUP.

As previously discussed, the pseudogestational sac sign has been reported in the setting of ectopic pregnancy. However, identifying a structure that may be a pseudogestational sac can be a pitfall, as this may represent a very early gestational sac instead. One needs to be very careful before concluding that an ectopic pregnancy is present based solely on the presence of a presumed pseudogestational sac, as incorrect diagnosis may result in administration of cytotoxiv therapy that can have disastrous consequences for an intrauterine pregnancy. Careful clinical management and close imaging follow-up should be used if an ectopic is suspected and a pseudogestational sac sign is an isolated imaging abnormality.

195
Q

Ectopic locations

A

Most (~95%) ectopic pregnancies occur in the fallopian tube, with the ampullary segment being the most common iste.

It is rare for an ectopic pregnancy to be in the ovary (< 1% of all ectopics).

An especially dangerous location for an ectopic pregnancy is the interstitial portion of the fallopian tube. An interstitial ectopic carries an especially high risk of catastrophic hemorrhage due to its propensity for delayed rupture and proximity to the ovarian vessels. Ultrasound of an interstitial ectopic shows absent myometrium along the lateral edge. The interstitial line sign represents a thin, echogenic line extending from the endometrial canal to the center of the interstitial ectopic mass. This echogenic line is thought to represent the nondistended, empty endometrial canal.

A heterotopic pregnancy is a simultaneous IUP and ectopic pregnancy. Patients undergoing assistive reproductive techniques are at increased risk for heterotopic pregnancy. Prior to the popularity of these fertilization procedures, heterotopic pregnancies were extremely rare. Given the increased prevalence of assistive reproductive techniques, however, the adnexae must be carefully evaluated even in the presence of an IUP.

Ectopic pregnancy may also occur (rarely) in a prior Cesarean section scar, in the cervix, or in the abdomen. When in the abdomen, the ectopic prenancy can become large before causing symptoms.

196
Q

Imaging findings of ectopic pregnancy

A

In the absence of an IUP, an adnexal mass has a 95% positive predictive value of being an ectopic.

An extrauterine embryo (with or without heartbeat) or mass with yolk sac has a 100% predictive value of being an ectopic.

The nonspecific ring of fire sign describes increased peripheral color Doppler flow surrounding an adnexal mass. This sign is rarely helpful as it can be seen in both ectopic pregnancy and corpus luteum.

In a rule-out ectopic patient, an ovarian mass is overwhelmingly more likely to be a corpus luteum than an ectopic pregnancy unless a definite embryo or yolk sac is identified. In contrast, an extra-ovarian mass would be concerning for ectopic. In ambiguous cases, it may be helpful to gently apply pressure with the ultrasound probe and watch for movement of the mass with respect to the ovary to identify if a mass is ovarian or adnexal in origin.

The pattern of hCG levels over time may be a helpful adjunct to imaging in following up the rule-out ectopic patient.

197
Q

Gestational trophoblastic disease (hydatidiform molar pregnancy)

A

Gestational trophoblastic disease, also called hydatidiform molar pregnancy (or molar pregnancy for short), is invasive neoplastic overgrowth of the trophoblast into the myometrium or beyond. The trophoblast normally develops into the placenta.

The classic clinical presentation of molar pregnancy is hyperemesis, markedly elevated hCG, and an enlarged uterus. The patient may also present with painless vaginal bleeding.

Complete hydatidiform mole does not contain any fetal parts. It is caused by loss of the egg’s DNA prior to fertilization by the sperm and has a diploid karyotype of 46,XX (most commonly) or 46, XY. A complete mole may pregress to metastatic choriocarcinoma. Chorioadenoma destruens is a complete mole that invades the myometrium. Molary pregnancy is associated wtih theca lutein cysts, which arise in response to elevated hCG.

Partial hydatidiform mole is associated with some fetal development. It is usually caused by two sperm fertilizing the same egg and has a triploid karyotype of 69, XXX, 69, XXY, or 69, XYY. Partial mole is less likely to progress to choriocarcinoma.

On ultrasound, molar disease causes uterine enlargement with a classic heterogenous and multicystic snowstorm appearance. Visualization of fetal parts suggests a partial mole.

Treatment of a molar pregnancy is endometrial suction currettage and close follow-up of serum hCG levels.

198
Q

Retained products of conception (RPOC)

A

Retained products of conception (RPOC) are placental or fetal tissues that remain in the uterus after delivery, miscarriage, or termination.

Untreated RPOC can lead to continued maternal bleeding and endometritis.

The sonographic findings of an endometrial blood clot and RPOC overlap and it is often not possible to differentiate between these two entities. An endometrial mass, with or without Doppler flow, in the appropriate clinical context has only about a 50% positive predictive valuse. The presence of Doppler flow is not diagnostic of RPOC, as endometrial color Doppler flow can be detected in the presence or absence of RPOC. Color Doppler flow is seen more commonly with RPOC, however.

A normal-appearing uterus, with an endometrial thickness less than 10 mm, is highly unlikely to contain RPOC.

199
Q

Multiple gestations and placentation

A

The placentation type (chorionicity and amnionicity) substantially affects the risk for pregnancy complications and influences how closely the pregnancy should be followed. The placentation should always be stated when first describing a multiple gestation.

The zygosity (number of fertilized eggs) cannot always be determined by ultrasound. Monozygotic twins can have any placentation type, depending on when the developing zygote splits. Dizygotic twins, however are always diamniotic/dichorionic, and only dizygotic twins can be different sexes. Monozygotic (“identical”) twins arise from a single egg fertilized with a single sperm. Dizygotic (“fraternal”) twins arise from two individually fertilized eggs.

The chorionicity is the number of placentas. Monochorionic twins share a single placenta. Dichorionic twins each have a separate placenta.

Amnionicity is the number of amnions. Monochorionic twins share a single amniotic sac. Diamniotic twins each have a separate amniotic sac.

By convention, chorionicity is stated before the amnionicity when starting the placentation. For instance, the abbreviation mono/di refers to monochorionic/diamniotic twins.

200
Q

Overview of complications by placentation type

A

The more that the twins share, the greater the risk of complications.

Di/di twins have an increased risk of premature delivery and low birth weight compared to singleton gestations.

Mono/di twins have an increased risk of complications related to a shared placenta including twin-twin transfusion, acardiac twin syndrome, and twin embolization.

In addition to being at risk for the same complications as mono/di twins, mono/mono twins are also at risk for cord entanglement and being conjoined.

201
Q

Early counting of multiple gestations

A

If two separate gestational sacs are identified, the placentation is di/di. The zygosity is indeterminate. Although dizygotic twins are always di/di, early splitting of a single fertilized egg can also lead to di/di monozygotic twins.

If a single gestational sac contains two yolk sacs, the placentation is mono/di, and the twins are monozygotic.

202
Q

Dizygotic (“fraternal”) twins

A

Dizygotic twins result when two eggs are each fertilized by a different sperm.

Dizygotic twins are always dichorionic/diamniotic.

203
Q

Monozygotic (“identical”) twins

A

Monozygoitc twins are the result of the blastocyst or embryo, formed by fertilization of a single egg by a single sperm. Although “identical” monozygotic twins are always the same sex, they may not be identical phenotypically due to local differences in the uterine and placental environment.

33% split early (0-4 days), before formation of either the placenta or amnion, leading to dichorionic/diamniotic twins.

66% split intermediate (4-8 days), after formation of the placenta but before the amnion has developed, leading to monochorionic/diamniotic twins.

1% split late (>8 days), after formation of the chorion and amnion, leading to monochorionic/monoamniotic twins. Very late splitting may result in conjoined twins, which are always monochorionic/monoamniotic.

204
Q

Di/di (dichorionic/diamniotic) twins

A

Di/di twins each have a separate placenta and amniotic sac.

On ultrasound, two placentas can usually be separately identified.

The inter-twin membrane will be relatively thick as there are two layers of chorion and two layers of amnion separating each twin.

In the second and third trimesters, the thickness of the inter-twin membrane is less reliable to determine chronicity because the membrane becomes thinner has gestation progresses.

The twin peak sign (also called the lambda sign) represents a triangle-shaped placental infolding at the interface of the placenta and the thick inter-twin membrane that is seen in di/di twins. The twin peak sign is most useful when it is difficult to distinguish two placentas.

If the twins are different sexes, they must be dizygotic twins, which are always di/di.

205
Q

Mono/di (monochorionic/diamniotic) twins

A

Mono/di twins share a placenta, but have separate amniotic sacs.

The shared placenta is usually apparent on ultrasound.

The inter-twin membrane is thin, as it is composed of only two layers of amnion.

206
Q

Mono/mono (monochorionic/monioamniotic) twins

A

Mono/mono twins share both a placenta and a single amniotic sac.

Mono/mono twins have a shared palcenta with no intervening membrane between the twins. Intertwined cords are diagnostic of mono/mono placentation when seen.

Isolated lack of visualization of an inter-twin membrane is not sufficient to diagnose mono/mono twins. If no inter-twin membrane (or intertwining cord) is seen, then the amnionicity cannot be determined. Because mono/mono twins are so rare, it is more common to have mono/di twins with non-visualization of the inter-twin membrane.

207
Q

Conjoined twins

A

Conjoined twins are caused by late (>13 days) incomplete division of the embryo.

208
Q

Twin-twin transfusion syndrome (TTTS)

A

Twin-twin transfusion syndrome (TTTS) is a complication of monochorionic twins (either mono- or di-amniotic) caused by disproportionite blood flow between the fetuses.

The donor twin transfers excess blood flow to the recipient twin. The donor twin is small and has oligohydramnios. The recipient twin is larger and has polyhydramnios.

There are three criteria to diagnose TTTS by ultrasound: 1) Disproportionate fetal sizes, with at least 25% discrepancy. 2) Disproportionate amniotic fluid, with the small twin having oligohydramnios and the large twin having polyhydramnios. 3) Single shared placenta (monochorionic).

There is a spectrum of severities of TTTS. In the earliest stages, the donor twin’s bladderis still visible and the direction of umbilical artery Doppler flow is normal. Later stages are marked by fetal hydrops or death.

A stuck twin describes severe oligohydramnios in the donor (small) twin. A stuck twin has so little amniotic fluid that the amnion is wrapped around the twin like shrink wrap.

Treatment options of TTTS include laser ablation of placental arteriovenous fistulas, therapeutic amniocentesis from the recipient (large, poly) twin, or selective coagulation of the umbilical cord of the less viable twin.

209
Q

Acardiac twins

A

Acardiac twinning, also called twin reversal arterial perfusion (TRAP) sequence, is a severe variant of twin-twin perfusion syndrome. Similar to TTTS, acardiac twinning is a complication of monochorionic twins (either mono- or di-amniotic).

In acardiac twins, the donor fetus supplies circulation to itself and an acardiac twin, enabled by placental fistulous connections. The acardiac twin has rudimentary or no development of structures above the thorax.

Doppler of the umbilical arteries and vein shows reversed flow in the acardiac twin. Normally, the umbilical arteries carry deoxygenated blood out of the fetus, pumped by the fetal heart. In the acardiac twin, the umbilical arteries carry blood out of the fetus, pumped by the fetal heart. In the acardiac twin, the umbilical arteries carry nutrient-depleted, poorly oxygenated blood into the fetus, pumped by the donor twin’s heart. Doppler of the acardiac twin’s umbilical arteries show an arterial waveform going into the fetus. Normally, the umbilical vein carries oxygenated blood into the fetus, from the placenta. In the acardiac twin, the umbilical vein carries deoxygenated blood out of the fetus. Doppler of the acardiac twin’s umbilical vein shows a venous waveform going out of the fetus.

Treatment is coagulation of the acardiac twin’s umbilical cord.

210
Q

Twin embolization syndrome

A

When one monochorionic twin dies in utero, the surviving twin is at risk for twin embolization syndrome, which can cause CNS, gastrointestinal, or renal infarcts.

In general, prognosis for a surviving monochorionic twin is very poor whne one twin dies in utero. In contrast, prognosis is generally good for a surviving dichorionic twin when one twin dies in utero.

211
Q

Crown-rump length

A

The crown-rump length (CRL) is used to assign gestational age from 6-`12 weeks. Measuring the CRL is straightforward in the first trimester as the fetus cannot flex or extend the neck.

212
Q

Prosencephalon and rhomboencephalon

A

By 8 weeks, the forebrain (prosencephalon) can be distinguished from the hindbrain (rhomboencephalon). Both prosencephalon and rhombencephalon are hypoechoic although the rhombencephalon is much more prominent. Absence of these structures may be the earliest finding of anencephaly.

213
Q

Ventral abdominal wall

A

The midgut normally herniates through the ventral abdominal wall in the first trimester. During this herniation, the midgut rotates 270 degrees around the axis of the superior mesenteric artery (SMA).

Physiologic midgut herniation is usually complete by 12-13 weeks. Therefore, a pathologic ventral wall defect, such as omphalocele or gastroschisis, is generally not diagnosed before 13 weeks.

It is common to see some fullness at the base of the umbilical cord before 13 weeks, which usually represents physiologic midgut herniation. If the fullness is especially prominent then it may be prudent to bring the patient back for a follow-up at 13 weeks to evaluate for a true ventral wall defect.

214
Q

Nuchal translucency

A

Increased thickness of the nuchal translucency is associated with increased risk of Down syndrome and other chromosomal abnormalities. With a fixed false-positive rate of 5%, nuchal translucency alone can detect approximately two thirds of cases of trisomy 21.

The nuchal lucency must be measured properly to obtain an accurate value.

At 11 weeks, the upper limit of normal is 2.2 mm.

At 14 weeks (CRL 79mm), the upper limit of normal is 2.8 mm.

Nucal translucency is combined with maternal serum testing to calculate an overall risk of trisomy 21.

215
Q

Head measurements

A

The biparietyal diameter (BPD) is meaasured from the outer edge of the skull closest to the transducer to the inner edge of the skull farthest from the transducer. The plane of measurement is at the level of the talamic and cavum septum pellucidum. The skull should be completely visualized all the way around. The corrected BPD incorporates the occipital frontal diameter (OFD) and a correction factor.

The occipital frontal diameter (OFD) is measured from the middle of the frontal skull to the middle of the occipital skull. The measurement plane is the same as that used to measure the BPD, at the level of the thalami and cavum septum pellucidum.

216
Q

Abdomen measurements

A

The abdominal diameter is measured from outer skin-to-skin in AP and transverse at the level of the intrahepatic umbilical vein, portal vein, and fetal stomach.

Ideally, the abdomen should be round, with less than 1 cm difference between the AP and transverse measurements.

The entire circumference of the skin should be well visualized.

The best measurements are often obtained if the anterior-posterior axis of the fetal abdomen is angled approximately 45 degrees so that the artifacts from the spine are minimized.

217
Q

Femur length

A

The femur length is most accurately measured when the femur is closest to the transducer, perpendicular to the sound beam.

218
Q

Amniotic fluid index (AFI)

A

To quantify the amniotic fluid index (AFI) between 16 and 42 weeks, the largest vertical pocket of fluid is measured (in cm) in each of the four quadrants and summed. AFI varies with gestational age. In borderline cases, the subjective assessment should take precedent. Some references state an AFI between 7 to 25 is normal, but these cutoffs vary. Oligohydramnios: AFI < 6.3cm is < 2.5th percentile. Peaks at 24 weeks: 9.0 cm = 2.5th percentile. Polyhydramnios: AFI >19.2 cm is >97.5th percentile. Peaks at 36 weeks: 27.9 cm = 97.5th.

The amount of amniotic fluid should always be subjectively assessed.

219
Q

Nuchal fold (second trimester only)

A

A thickened nuchal fold is the most sensitive and specific ultrasound finding to suggest Down syndrome.

Compared to nuchal lucency, measurement of nuchal fold is performed later in pregnancy. In contrast to the nuchal lucency, the nuchal fold is measured in the axial plane at the level of the posterior fossa.

The nuchal fold is only measured from 16-20 weeks <5mm is normal. 5-5.9 mm is borderline, >6 mm is a major marker for trisomy 21.

A very thick nuchal fold may represent a cystic hygroma, which is associated with Turner syndrome (45, X).

220
Q

Cervical shortening

A

Shortening of the cervix is a risk factor for pre-term delivery. A cervical length <3 cm is abnormal. The presence of cervical funneling (change in shape) is an ancillary finding. The mneumonic trust your vaginal ultrasound can be used to remember the sequence of cervcial funneling. A T-shaped cervix is normal. As funneling progresses, the cervix resembles Y, V, and U shapes.

Prior to viability (24 weeks), treatment is cervical cerclage. After 24 weeks, treatment tends to be conservative (bedrest) due to concern for membrane rupture with any procedure.

221
Q

Placental embryology and physiology

A

The placenta is formed by fetal chorion and maternal endometrium.

The mature placental circulation allows exchange of oxygen and nutrients between maternal and fetal vessels through a membrane, although the blood does not admix.

222
Q

Single umbilical artery

A

The normal umbilical cord has two umbilical arteries and a single umbilical vein. A single umbilical artery is associated with fetal anomalies (most commonly cardiovascular) in up to 50% of fetuses.

There is an increased incidence of a single umbilical artery in trisomies 13 and 18.

223
Q

Abnormalities of placental thickness

A

Placental thickness is not routinely measured, but in fetal hydrops the placenta may become thickened.

In polyhydramnios, the placenta usually becomes stretched and thinned. In the presence of polyhydramnios, if the placenta looks normal in thickness, or especially if the placenta looks thickened, concern should be raised for fetal hydrops.

224
Q

Vasa previa

A

Vasa previa is the traversing of fetal placental vessels across the internal cervical os, which can be caused by velamentous insertion or a placental succenturiate lobe. Velamentous insertion is the insertion of the umbilical cord outside the margin of the placenta. A succenturiate lobe is an island of placental tissue separate from teh main placenta, connedted to the main placenta by blood vessels.

225
Q

Placenta previa and other abnormalities of placental position

A

Obstetrical terminology regarding abnormalities of placental position in relation to internal cervical os is confusing. The terms partial previa, complete previa, incomplete previa, marginal placenta, and low-lying placenta are inconsistent among references.

The edge of the placenta should be >3 cm from teh internal cervical os.

If the placenta is <3 cm from the internal cervical os, it is best to simply be descriptive and describe how far the leading edge of the placenta is from the internal cervical os.

In true placenta previa, the placenta covers the internal cervical os. Placenta previa is seen in approximately 0.5% to 1% of deliveries and requires a Cesarean section for safe delivery. A potential pitfall is over-diagnosis in the first and second trimester due to contractions or overfilling of the maternal bladder. To definitively diagnose previa, the patient should have an empty bladder and realtime scanning should be performed to confirm the lack of active contractions.

226
Q

Placental abruption

A

Placental abruption is premature separation of the placenta from its uterine attachment.

There is an increased incidence of abruption in maternal hypertension, drug abuse, trauma, or rapid decompression of a distended uterus (e.g., from a large-volume amniocentesis).

A subchorionic hematoma is a variant seen early in pregnancy, where a hypoechoic crescent is present around the gesational sac.

Placental abruption can have a variable ultrasound findings and may even appear normal. Because a normal ultrasound is seen in 20% of cases of abruption, a negative ultrasound cannot exclude abruption. Typically, ultrasound of abruption shows a placental hematoma, whih can be subchorionic (most commonly), retroplacental, or pre-placental. On clinical exam, blood may be present in the vaginal canal.

Acute abruption may be especialy challenging to diagnose on imaging as the hematoma is isoechoic to placenta.

Chronic abruption features hypoechoic blood products wihtin the placenta.

227
Q

Placenta accreta

A

Placenta accreta is a spectum of abnormally tenacious or deep attachment of the placenta into the myometrium, carrying a risk of hemorrhage at the time of placental separation.

Accreta is thought to be caused by scarring (which may be secondary to prior Caesarean section, D&C, endometritis, or adenomyosis) and resultant endometrial deficiency. It is especially important to consider accreta if an anterior placenta is present with a history of prior Caesarean section. Placenta previa also increases the risk of accreta.

Ultrasound findings of accreta include loss of the normal retroplacental clear space, abnormalities at the bladder/placental interface, and prominent vascular lacunar spaces. The presence of a moth-eaten placenta with vascular spaces near the bladder is highly specific for accreta.

Placenta accreta is an umbrella term that describes three degrees of placental attachment/invasion. Confusingly, the term “placenta accreta” also describes one of the specific three degrees.

In placenta accreta, the placenta attaches deeply into the myometrium but does not invade. Ultrasound shows thinnign or absence of the normal hypoechoic subplacental zone.

In placenta increta, the placenta invades the myometrium.

In placenta percreta, placenta penetrates through the myometrium and into or through the serosa.

228
Q

Overview of fetal hydrops

A

Hydrops is a fluid-overload state characterized by at least two of the following: Ascites, pleural or pericardial effusion, skin thickening, polyhydramnios, and placental enlargement.

Hydrops may be classified as immune or non-immune. Prognosis is variable but tends to be poor for non-immune hydrops.

229
Q

Immune hydrops

A

Immune-mediated hydrops is fetal hemolytic anemia caused by prior maternal exposure to fetal antigens, by far most commonly the Rh antigen.

Prognosis is good if treated with intrauterine or peripartum fetal blood transfusions.

230
Q

Non-immune hydrops

A

Non-immune hydrops can be due to a diverse array of causes, most of which lead to a common pathway of extracellular fluid overload. Prognosis of non-immune hydrops tends to be poor, as the primary cause is often not effectively treatable.

Etiologies include (more common causes in bold): Primary cardiac abnormalities, including structural abnormalities and arrhythmias. Extra-cardiac shunt, including vein of Galen malformation, hepatic hemangioendothelioma, and twin-twin transfusion syndrome, all of which may lead to high-output cardiac failure. Infectious, especially parvovirus B19 and TORCH infections. Decreased oncotic pressure, due to hepatitis and fetal nephrotic syndrome. Increased capillary permeability, which can be due to anoxic injury. Venous obstruction, seen in Turnder (45, X) syndrome.

231
Q

Fetal ascites

A

When seen with other abrnomalities, ascites is one of the criteria for diagnosis of hydrops.

Isolated fetal ascites may be due to urinary obstruction and resultant calyceal or bladder rupture or meconium peritonitis.

232
Q

Fetal pleural effusion

A

Fetal pleural effusion is a criterion for diagnosis of hydrops.

When seen in isolation (not a component of hydrops or due to any other abnormality), fetal pleural effusion is mostly commonly caused by congential chylothorax. Fetal chylothorax is thought to be due to thoracic duct or lymphatic malformation. At birth the pleural fluid is simple, but after the baby begins to drink milk it becomes chylous.

Fetal pleural effusions can also be seen in Turner or Down syndromes.

233
Q

Overview of fetal amniotic fluid

A

Amniotic fluid surrounds the fetus and is required for normal development of multiple organ systems, including the lungs.

The amount of amniotic fluid should routinely be assessed subjectively. It is not necessary to routinely measure the amount of amniotic fluid; however, if a measurement is needed, the amniotic fluid index (AFI) may be used for quantification.

Amniotic fluid is produced primarily by the fetal genitourinary tract and is excreted by the fetus as urine. A small amount of amniotic fluid is also produced by the fetal lungs and nasopharyngeal cavities.

Disorders of the fetal genitourinary tract may cause oligohydramnios due to insufficient secretion of fluid.

Amniotic fluid is absorbed primarily by fetal swallowing.

Disorders of the gastrointestinal tract or central nervous system may cause polyhydramnios due to impairment of swallowing or absorption of fluid.

234
Q

Oligohydromanios

A

Oligohydramnios is too little amniotic fluid.

Oligohydramnios may lead to Potter sequence, which describes the typical malformations induced by confinement by oligohydramnios including facial dysmorphism, pulmonary hypoplasia, club feet, and musculoskeletal contractures.

Most commonly, oligohydramnios is associated with intrauterine growth restriction (IUGR) without a fetal structural anomaly.

Although malformations are relatively uncommon, the genitourinary system must be carefully evaluated in the setting of oligohydramnios. Several genitourinary anomalies may lead to oligohydramnios, including: Renal agenesis - fatal if bilateral. Congenital bladder outlet obstruction, including posterior urethral valves. Bilateral ureteropelvic junction obstructions. Renal dysplasia, including autosomal recessive polycystic kidney disease (ARPKD).

235
Q

Polyhydramnios

A

Polyhydramnios is too much amniotic fluid.

Greater than half of cases of polyhydramnios are idiopathic, with a normal fetus. The remainder may be associated with chromosomal abnormalities, diabetes, or structural defects (primarily of the gastrointestianal tract), including: Primary upper GI obstruction or atresia, such as laryngeal, esophageal or duodenal atresia. Secondary obstruction, due to diaphragmatic hernia, gastroschisis, or omphalocele. Severe CNS anomalies (which often cause disorders in swallowing). Monochorionic twin syndromes, such as twin-twin transfusion syndrome. Placental abnormalities, such as chorioangioma. Chorioangioma is a benign placental hemangioma that may cause polyhydramnios when highly vascular.

236
Q

Ventriculomegaly

A

Ventriculomegaly is enlargment of the cerebral ventricles. The term hydrocephalus is usually avoided because that implies ventriculomegaly due to obstruction.

Throughout gestation, the lateral venticles should each measure less than 10 mm when measured at the atrium. The atrium is the confluence of the lateral ventricles, temporal horn, and occipital horn. The normal choroid pelxus has a rounded border in this location. Mild ventriculomegaly: 10-12 mm; moderate: 12-15 mm: marked: >15 mm

Normally, the choroid plexus fills the lateral ventricle. The dangling choroid sign represents the dependent drooping of choroid plexus seen in ventriculomegaly. Ventriculomegaly may be present even in a ventricle measuring <10 mm if there is >3 mm of fluid between the medial margin of the ventricle and the choroid.

Ventriculomegaly is a sign that something else is wrong, with a diverse array of etiologies: Primary CNS structural (aqueductal stenosis, Dandy Walker, Chiari II, holoprosencephaly, agensis of the corpus callosum). Genetic (trisomies 13 and 18). Destructive (due to infection, hemorrhage, or infarct). Idiopathic.

237
Q

Anencephaly

A

Anencephaly is a lethal anomaly with complete lack of development of fetal cerebral cortex and calvarium above the orbits.

AFP is elevated in anencephaly, as there is direct exposure of neural tissue to the amniotic fluid.

Anencephaly may cause polyhydramnios due to impairment in swallowing.

Angiomatous stroma is residual neural-type tissue that may be tethered above the head and may be confused with an encephalocele.

The differential of anencephaly is amniotic band syndrome, which is almost always asymmetric.

238
Q

Cephalocele

A

A cephalocele is a midline neural tube defect characterized by protrusion of intracranial structures outside of the calvarium. The occipital skull is the most common location.

A meningocele contains only meninges. An ecephalocele also contains neural tissue.

In addition to cephalocele, the primary differential consideration of a mass posterior to the occipital skull is a cystic hygroma, which is a congenital lymphatic malformation and the most common fetal neck mass.

239
Q

Dandy Walker malformation

A

Dandy Walker is a diverse spectrum of diseases characterized by hpogenesis of the cerebellar vermis and resultant fourth ventricular dilation.

240
Q

Chiari II/Myelomeningocele

A

Chiari II is the combination of a small posterior fossa and a neural tube defect. By far the most common associated neural tube defect is a lumbar myelomeningocele. A myelomeningocele contains both neural elements and meninges.

The banana sign describes the characteristic flattened cerebellar hemispheres in the small posterior fossa. The banana sign is very specific for Chiari II. In fact, if the banana sign is seen, then a myelomeningocele is presumed to be present even if not identified on ultrasound.

The lemon sign describes flattening of the frontal bones, causing the calvarium to have the morphology of a lemon when seen axially. Unlike the banan sign, the lemon sign is not specific for Chiari II.

241
Q

Holoprosencephaly

A

Holoprosencephaly is failure of midline cleavage of the primitive prosencephalon in early embryologic development. The most severe form, alobar holoprosencephaly, leads to fused thalami and a single monoventricle that may communicate with a large dorsal cyst. Brain tissue surrounds the monoventricle, forming a characteristic boomerang shape.

Holoprosencephaly is associated with trisomy 13, facial hypoplasias, and midline facial anomalies including clefts.

242
Q

Agenesis of the corpus callosum

A

Absence of the corpus callosum can be a difficult diagnosis ot make. Because the normalk corpus callosum is not always visuzualized on ultrasound, it is often necessary to rely on secondary abnormal morphology of the ventricular system to diagnose absence of the corpus callosum.

Absence of the cavum septum pellucidu (CSP) is associated with agenesis of the corpus callosum.

Abnormal teardrop morphology of the lateral ventricles with dilated occipital horns, known as coplocephaly, is asscoited with absence of the corpus callosum. Colpocephaly is often seen together with ventriculomegaly.

Widely separated ventricular frontal horns and parallel configuration of the lateral ventricles both suggest agenesis of the corpus callosum.

The medial borders of the lateral ventricles may be concave, due to protrusion of Probst bundles and the cingulate gyrus. Probst bundles are axons that normally constitue the corpus callosum. In agenesis of the corpus callosum, the Probst bundles do not cross the midline. Instead, they pursue an aberrant course parallel to the interhemispheric fissure, indenting the medial margin of the lateral ventricles.

A midline interhemispheric cyst may be present, representing superior herniation of the third ventricle.

A minority of babies with agenesis of the corpus callosum have trisomy 8, 13, or 18.

243
Q

Absence of the cavum septum pellucidum (CSP)

A

The cavum septum pellucidum (CSP) should always be identified in a normal fetus.

If the CSP is not seen, the primary consideration is agenesis of the corpus callosum, as the cavum septum pellucidum and corpus callosum are formed simultaneously.

Uncommonly, the CSP may be absent in the presence of a normal corpus callosum. This may represent septo-optic dysplasia and fetal MRI should be recommended.

244
Q

Hydrancephaly

A

Hydranencephaly is complete crotical destruction due to infarct or infection. The brain parenchyma is obliterated and replaced by fluid.

The most common cause of hydrancephaly is in utero complete occlusion of the MCA.

In contrast to severe hydrocephalus, a cortical mantle is absent in hydrancephaly.

In contrast to holoprosencephaly, a falx is typically visualized in hydrancephaly.

245
Q

Choroid plexus cyst

A

Cysts within the choroid plexus are common (seen in 1-5% of fetal surveys). The vast majority of choroid plexus cysts are present in normal fetuses and resolve on follow-up scans. However, up to 50% of trisomy 18 fetuses will have choroid plexus cysts.

Choroid plexus cysts can be considered an incidental finding in the absence of any other sonographic abnormality and with a normal maternal serum screen.

A potential mimicker of a choroid plexus cyst is the spongy choroid, which describes a heterogenous echogenic choroid that is a normal variant.

246
Q

Vein of Galen malformation

A

Vein of Galen malformation is dilateion of the vein of Galen (in the pineal region) caused by an arteriovenous fistula.

Vein of Galen aneurysm is a shunt lesion that may cause high-output cardiac failure, leading to hydrops.

247
Q

Congenital diaphragmatic hernia (CDH)

A

Congenital diaphragmatic hernia (CDH) is herniation of abdominal organs (most commonly bowel) into the thorax through a diaphragmatic defect. CDH is the most common fetal intrathoracic mass lesion.

Most cases are isolated anomalies; however, a prominent minority of fetuses have other anomalies, most commonly congenital heart disease.

By far, the most common location for CDH is the left posterior thorax, called a Bochdalek hernia (mnemonic: “back to the left”). A Bochdalek hernia may displace the heart.

When a CDH occurs on the right it is called a Morgagni hernia. The diaphragmatic defect of a Morgagni hernia tends to be anterior, with the liver the most commonly herniated organ.

The two classic fetal ultrasound findings of CDH are a cystic intra-thoracic mass representing the stomach and/or bowel, and absence of the stomach bubble below the diaphragm.

Complications of CDH include pulmonary hypoplasia on the affected side, bowel obstruction with resultant polyhdramnios, and obstruction of venous return due to IVC compression, which may lead to ascites.

248
Q

Bronchopulmonary foregut malformation

A

Bronchopulmonary foregut malformations are a spectrumn of congenital abnormalities of the fetal lungs and upper GI tract.

Congenital pulmonary airway malformation (CPAM) is a hamartomatous proliferation of small airways that communicates with the bronchial tree. Blood supply is from teh pulmonary circulation. CPAM was previously called congenital cystic adenomatoid malformation (CCAM). CPAM can be classified into three types based on the size of cysts (Type 1 - large cysts; Type II - small cysts; Type III - tiny cysts too small to see on ultrasound). Today this classification is not used much because prognosis is dependent on size of the entire lesion rather than the size of the individual cysts. CPAM is not associated with other anomalies (unlike CDH). CPAM may appear to spontaneously regress on ultrasound, but will remain apparent on CT or MRI.

Sequestration is aberrent lung tissue with a systemic blood supply, usually from the aorta. The most characteristic location of sequestration is the left lower lobe. The classic ultrasound appearance of sequestration is an echogenic mass at the left lung base. The sequestration may occasionally be subdiaphragmatic and simulate an adrenal mass. Systemic blood supply should be confirmed with color Doppler. In the absence of the Doppler findings, sequestration may be difficult to differentiate from CPAM. In contrast to the findings of CPAM, cysts are less common, but there is less mass effect, and location is almost always in the lower lobes.

Bronchogenic, gastrointestinal duplication, and neurenteric cysts almost always appear as solitary, simple cysts on ultrasound.

249
Q

Pulmonary hypoplasia

A

Pulmonary hypoplasia is inadequate lung development. Hypoplasia can be due to a thoracic mass lesion (such as CDH), oligohydramnios, or a skeletal dysplasia affecting the ribs.

It is important to evaluate the size of the fetal thorax in relation to the abdomen on coronal images. A small, bell-shaped thorax suggests pulmonary hypoplasia.

250
Q

Laryngeal or tracheal atresia

A

Atresia of the upper airway, otherwise known as congenital high airway obstruction syndrome (CHAOS), is lethal and may cause bilateral enlarged echogenic lungs.

251
Q

Esophageal atresia

A

Esophageal atresia is a blind-ending esophagus, due to incomplete division of the foregut in early embryologic development.

Esophageal atresia is usually associated with a tracheoesophageal fistula.

The classic ultrasound finding of esophageal atresia are polyhydramnios and an absent stomach bubble.

252
Q

Duodenal atresia (DA)

A

Duodenal atresia (DA) causes duodenal obstruction from lack of recanalization of the duodenal lumen. Duodenal atresia is the most common cause of fetal duodenal obstruction.

Approximately one third of babies with DA will have Down syndrome. If a double bubble sign is seen, a careful screen for additional findings of Down syndrome should be performed (e.g., thorough heart exam, nuchal fold if 16-20 weeks, etc.).

The classic appearance of DA is the double bubble sign, representing a dilated stomach and dilated proximal duodenum. The differential diangosis of the double bubble sign includes duodenal web, stenosis, and annular pancreas.

253
Q

Distal fetal bowel obstruction

A

A distal fetal bowel obstruction may be structural or functional.

Structural causes of distal bowel obstruction include jejunal atresia, ileal atresia, and anorectal malformation. Anorectal malformation is commonly associated with additional abnormalities, including the VACTERL association (vertebral, anorectal, cardiac, tracheoesophageal, renal, and limb anomalies).

Functional bowel obstruction may be due to Hirschsprung disease or meconium ileus. Hirschsrpung disease results in a functional obstruction due to absent distal enteric ganglion cells. Meconium ileus causes obstruction from impaction of meconium in the ileum. Nearly all infants with meconium ileus have cystic fibrosis; however, meconium ileus is rarely identified before the 3rd trimester.

254
Q

Fetal manifestations of meconium

A

Meconium ileus is bowel obstruction caused by impacted meconium in a fetus with cystic fibrosis.

Meconium peritonitis is peritoneal inflammation secondary to in utero bowel perforation and resultant spillage of meconium into peritoneal cavity. This leads to peritoneal adhesions and, ultimately, dystrophic calcifications.

Meconium pseudocyst is a cystic abdomina structure, often with peripheral calcifications, representing a walled-off bowel perforation. It is a sequela of meconium peritonitis.

255
Q

Hyperechoic small bowel

A

Hyperechoic or echogenic bowel is a nonspecific finding that is associated with Down syndrome. Other causes of hyperechoic small bowel include TORCH infection, cystic fibrosis, and swalloing of intra-amniotic blood. It may also be associated with intrauterine growth restriction.

If the bowel is only mildly echogenic (less echogenic than bone) and no mass effect is present, this appearance may represent a normal variant.

256
Q

Omphalocele

A

An omphalocele is a midline anterior abdominal wall defect with resultant herniation of intra-abdominal contents covered by a peritoneal membrane. Omphalocele is the most common anterior abdominal wall defect.

The key to differentiate omphalocele from gastroschisis is the position of the umbilical cord insertion. In omphalocele, the umbilical cord inserts centrally at the base of the herniated sac.

When small, omphaloceles often contain just bowel. Larger omphaloceles may also contain liver and carry a worse prognosis.

Omphalocele is associated with other anomalies in 50-75% of cases, including cardiac anomalies, trisomies, and Beckwith-Wiedemann syndrome (a congenital overgrowth syndrome characterized by omphalocele, macroglossia, hemihypertrophy, and visceromegaly).

257
Q

Gastroschisis

A

Gastroschisis is a paraumbilical (usually right-sided) anterior abdominal wall defect, through which bowel herniates without a peritoneal covering. Gastroschisis is the second most common abdominal wall defect.

Unlike omphalocele, gastroschisis is usually seen as an isolated anomaly.

Gastroschisis is seen more commonly in very young (teenage) mothers.

258
Q

Pentaology of Cantrell

A

Pentalogoy of Cantrell is a rare disorder consisting of ectopia cordis (extra-thoracic heart), omphalocele, diaphragmatic defect, pericardial defect, and disruption of the sternum.

259
Q

Fetal Genitorurinary tract

A

Although oligohydramnios may be due to many causes, the complete genitourinary (GU) tract (kidneys, ureters, bladder, and urethra) should be carefully evaluated in every fetus with oligohydramnios.

Oligohydramnios due to a fetal GU malformation can be divided into three categories: Fetal hydronephrosis (obstructive uropathy). Cystic renal disease. Bilateral renal agenesis.

If the fetal bladder is not visualized (a bladder not filled with fluid will not be visible on ultrasound) the cause of oligohydramnios is likely a renal anomaly, such as: Bilateral multicystic dysplastic kidneys. Bilateral renal agenesis. Autosomal recessive polycystic kidney disease (ARPKD)

Normal fetal kidneys grow approximately 1 mm per week of gestation. For instance, a 20 week fetus should have kidneys approximately 2 cm in length.

260
Q

Fetal hydronephrosis overview

A

Screening for fetal hydronephrosis is routinely performed to evaluate for potentially treatable causes, such as obstruction or reflux, which may lead to progressive childhood renal failure if undiagnosed.

The axial diameter of the renal pelvis is measured to assess for hydronephrosis. The accepted range of normal depends on gestational age. Less than 4 mm is always normal and >10 mm is always abnormal. Normal <5 mm greater than 20 weeks; <4 mm at 16-20 weeks. Equivocal: 5-9 mm greater than 20 weeks; 4-6 mm at 16-20 weeks. Abnormal: >10 mm at 30 weeks; >8 mm at 20-30 weeks; >6 mm at 16-20 weeks.

Hydronephrosis, hydroureter, and a normal bladder may be due to distal ureteral obstruction or reflux. Distal ureteral obstruction from ectopic insertion of the ureter into the lbader is often associated with the upper pole moiety of a duplicated collecting system. The most common ectopic insertion of an upper pole moiety is inferior and medial to the normally inserting lower pole ureter. A ureterocele is dilation of the intramural portion of the ureter which balloons out into the bladder and causes a functional obstruction at the ureterovesicular junction. Ureterocele is also associated with ectopic insertion of the upper pole moiety of a duplicated system. Although not a physical obstruction, severe vesicoureteral reflux may cause hydronephrosis and hydroureter, but the bladder remains normal.

Bladder outlet obstruction leads to hydronephrosis, hydroureter, and a dilated bladder. Posterior urethral valves is by far the most common cause of the bladder outlet obstruction. Urethral atresia is a rare cause of bladder outlet obstruction.

Hydronephrosis can lead to cystic renal dysplasia, a sign of irreversible renal damage.

261
Q

Posterior urethral valves

A

Posterior urethral valves is congenital obstruction of the posterior urethra due to a membranous flap in the proximal urethra.

Ultrasound shows severe dilation of the posterior urethra resulting in the keyhole sign. The bladder is typically enlarged and thickened. Hydroureteronephrosis and oligohydramnios are usually present.

262
Q

Autosomal recessive polycystic kidney disease (ARPKD)

A

Autosomal recessive polycystic kdiney disease (ARPKD) is a congenital disorder of diffuse collecting tubule dilation, leading to innumerable tiny renal cysts that are too small to be resolved by sonography.

Ultrasound findings include very large and echogenic kidneys and severe oligohydramnios, caused by markedly reduced renal function.

Prognosis is poor. ARPKD is associated with hepatic fibrosis if the baby survives infancy.

263
Q

Multicystic dysplastic kidney (MCDK)

A

Multicystic dysplastic kidney (MCDK) is thought to be the end result of fetal obstructive uropathy.

If MCDK is unilateral and there are no associated abnormalities, the prognosis is excellent. MCDK may be fatal if bilateral, however.

MCDK may affect only the upper pole of an obstructed duplicated system.

On imaging, multiple non-communicating cysts are interspersed with dysplastic renal parenchyma. In contrast to hydronephrosis, the cysts of MCDK do not connect to the collecting system.

The natural history of MCDK is gradual involution.

264
Q

Osteogenesis imperfecta (OI)

A

Osteogenesis imperfecta (OI) is a spectrum of congenital bone anomalies characterized by multiple fractures due to abnormal type 1 collagen. There are several types of OI, with type 2 being lethal. Type 2 can be diagnosed in the second trimester, but types 1, 3, and 4 are not typically diagnosed until the third trimester.

OI causes severe shortening of the long bones (<3 SD below the mean). The long bones and ribs appear “wrinkled” due to multiple fractures. The thorax is usually small due to broken and structurally soft ribs.

Unlike thanatotophoric dysplasia, bony mineralization is decreased. Decreased calvarial mineralization causes the entire brain (including the nearfield) to be visualized well.

265
Q

Thanotrophoric dysplasia

A

Thanatophoric dysplasia is a lethal skeletal dysplasia with characteristic telephone receiver femurs, severe limb shortening and bowing, and rib shortening.

Platyspondyly is characteristic, which is flattening of the ossified portions of the vertebral bodies.

The classic cloverleaf skull is caused by protrusion of the frontal and temporal lobes.

Bony mineralization is normal.

266
Q

Sacrococcygeal teratoma

A

Sacrococcygeal teratoma is a germ cell tumor of the sacrum. It often presents prenatally as a mixed solid and cystic complex mass.

Sacrococcygeal teratoma may be associated with high output cardiac failure.

The other primary differential consideration for a distal fetal spinal mass is a myelomeningocele, which is associated with spine abnormalities and a dorsal skin defect.

267
Q

Trisomies and syndromes

A

Fetal anomalies often occur in associations. For instance, omphalocele may be a sentinel finding that signifies the presence of trisomy 13, 18, or Beckwith-Wiedemann syndrome. If a sentinel finding is seen, a careful serach should be performed for associated anomalies seen in trisomies and syndromes.

268
Q

Trisomy 13 from head to toe

A

Holoprosencephaly and midline facial anomalies.

Encephalocele.

Congenital heart disease

Omphalocele

Horseshoe kidney

Polycystic kindeys

Polydactyly

269
Q

Trisomy 18 from head to toe

A

Strawberry sign: Inward bowing of the frontal bone creates a strawberry shape to the calvarium, with the tip of the strawberry projected anteriorly.

Choroid plexus cysts.

Facial cleft.

Micrognathia.

Cardiac anomalies

Omphalocele.

Congenital diaphragatic hernia.

Horseshoe kidney.

Hydronephrosis.

Clenched hand that never opens, with overlapping fingers.

Rocker bottom feet.

270
Q

Trisomy 21 (Down syndrome) from head to toe

A

Increase in nuchal fold (>6 mm), measured between weeks 15 and 21, is the single most sensitive and specific ultrasound finding for trisomy 21.

In contrast, nuchal transluceny is measured earlier in the pregnancy (11-14) weeks, and is less specfic for trisomy 21.

Absent ossification of nasal bone.

Cystic hygroma (although more common in Turner syndrome).

Congenital heart disease: VSD and endocardial cushion defect in particular.

Echogenic bowel.

Duodenal atresia.

Echogenic intracardiac focus.

Shortened femur and humerus.

Hypoplasia of the middle phalanx of the little finger.

Sandal gap toes.

271
Q

Beckwith-Wiedemann syndrome (BWS)

A

Beckwith-Wiedemann syndrome (BWS) is a syndrome of overgrowth that carries an increased risk of childhood cancers. BWS is mostly sporadic, but 10-15% of cases follow an autosomal dominant inheritance.

BWS increases the risk of developing Wilms tumor (the most common tumor in BWS), hepatoblastoma, and other childhood tumors. The standard of care is screening with abdominal ultrasound every 3 months until age 8.

Hemihypertrophy, organomegaly.

Macroglossia.

Omphalocele.

Perinatal hypoglycemia.

272
Q

Meckel-Gruber

A

Meckel-Gruber is an autosomal recessive multi-organ syndrome.

Encephalocele.

Renal dysplasia causing multiple tiny renal cysts, which appear as echogenic kidneys, analogous to ARPKD.

Polydactyly.

273
Q

Germinal matrix hemorrhage

A

The germinal matrix is the site of national precursor cells located in teh caudothalamic groove.

Grade I: Hemorrhage confined to the germinal matrix.

Grade II: Hemorrhage extends into the ventricles without ventriculomegaly.

Grade III: Hemorrhage extends into the ventricles with venticulomegaly.

Grade IV: Hemorrhage extends out of the ventricles into the parenchyma.